You are on page 1of 389

SLIDES

FOR
PEDIATRIC
OSCE EXAM
SYNDROMES
DYSMORPHIC DISORDERS
CONGENITAL MALFORMATION
SLIDES
1. William syndrome
2. Cornelia de Lange or Brachmann– de Lange syndrome
3. Down syndrome
4. Edward syndrome
5. lymphedema (turner syndrome)
6. carazon syndrome (pic of craniosyntosis and clinidactyly).
7. Hurler syndrome
8. Menkes disease
9. Noonan’s Syndrome
10. Klippel lindau syndrome
11. Beckwith-Wiedemann Syndrome (Rx refractory hypoglycemia)
12. Brune belly syndrome
13. Osteogenesis Imperfecta
14. achondroplasia
15. leish nodule- neurofibromatosis type 1 (NF1)
16. Kayser flisher ring / Wilson disease
17. cataract + renal invovment....lowes disease
18. Coloboma - CHARGE association
19. Abnormal red reflex -Leucocoria ,DDx
Slide 1

Turner syndrome newborn

 a webbed neck with low hairline


 shield chest with widespread nipples
 abnormal ears, and micrognathia in the left image
 prominent lymphedema of the hand in the right image.
Slide 2
Clinical features of Turner's syndrome

• Lymphoedema of hands and feet in neonate, which may persist


• Spooned shaped nails
• Short stature - cardinal feature
• Neck webbing or thick neck
• Wide carrying angle (cubitus valgus)
• Widely spaced nipples
• Congenital heart defects (particularly coarctation of the aorta, bicuspid valve ,AS)
• Delayed puberty
• Ovarian dysgenesis result in infertility, pregnancy may be possible (IVF) with donated ova
• Hypothyroidism
• Renal anomalies ; Horseshoe , ectopic kidneys ,Ureteropelvic obstruction
• Pigmented moles
• Recurrent otitis media
• Normal intellectual function in most pt.
Slide 3

1. what is the main abnormality you can find it in this patient?


2. name the test you want to confirm your diagnosis?

Answer
Down syndrome

1. CVS abnormality (VSD).


2. cytogenetic test
Characteristic clinical manifestations of Down's syndrome

immediate medical complications is - increased risk of duodenal atresia, CHD

Typical craniofacial appearance

 Round face and flat nasal bridge


 Upslanted palpebral fissures
 Epicanthic folds (a fold of skin running across the inner edge of the palpebral fissure)
 Brushfield spots in iris (pigmented spots) 70%
 Small mouth and protruding tongue
 Small low set ears
 Flat occiput and third fontanelle
Other anomalies

 Short neck
 Single palmar creases, incurved fifth finger and wide 'sandal' gap between toes
 Hypotonia
 Congenital heart defects (40%) AVSD (ECCD)
 Congenital GIT defects (6%) ,Duodenal atresia (30%), Hirschsprung's disease (3 %)
Later medical problems

 Delayed motor milestones


 Moderate to severe learning difficulties
 Small stature
 Increased susceptibility to infections
 Hearing impairment from secretory otitis media 75%
 Visual impairment from cataracts 3%, squints, myopia
 Increased risk of leukaemia and solid tumours
 Risk of atlantoaxial instability
 Hypothyroidism 15% and coeliac disease
 Epilepsy
 Alzheimer's disease
Slide 4

Trisomy 18..
 Typical profile reveals prominent occiput, micrognathia
 posteriorly rotated malformed auricles.
 Clenched hand showing typical pattern of overlapping fingers.
 Rocker-bottom feet.
Slide 5
Clinical features of trisomy 18..
 Microcephaly, Meningomyelocele , Anencephaly
 Low-set ear (faun-like with flat pinnae)
 hypertelorism , Microphthalmia ,Epicanthal folds ,Short palpebral fissures
,coloboma
 Short upturned nose, Choanal atresia
 Micrognathia/ retrognathia ,Microstomia
 Characteristic hand posture, with clenched hands with the index finger
overriding the middle finger and the fifth finger overriding the fourth finger, ,
Hypoplastic nails
 Syndactyly ,Clinodactyly
 Cardiac malformations in more than 90% of infants,VSD,ASD,PDA,TGA ,TOF
,HLHS
 Pulmonary hypoplasia
 Omphalocele ,Prune belly anomaly ,Umbilical, inguinal or diaphragmatic hernias
 Cryptorchidism, hypospadias and micropenis in males
 Horseshoe kidneys ,Ectopic kidney ,renal agenesis
 Rocker-bottom feet with prominent calcanei
Slide 6

Williams syndrome.

 Periorbital fullness
 epicanthal folds
 prominent lips
 Long philtrum
 stellate lacy iris pattern.
Hallmark features include

 supravalvular aortic stenosis


 hypercalcemia
 friendly personality
 connective tissue abnormalities.
Slide 7

Cornelia de Lange, or Brachmann– de Lange Syndrome


Facial features are quite distinctive and include
 long eyelashes; afine, almost “brushed-on” appearance of the
arch to the eyebrows; occasional synophrys due to hirsutism;
small nose with anteverted nostrils; long philtrum;
 downturned upper lip with cupid’s-bow shape; and micrognathia.
Extremities are notable for small hands and feet, and varying
abnormalities can include proximally placed thumbs.
Slide 8
Cornelia de Lange syndrome

Characteristic craniofacial appearance


a. Microcephaly (98%)
b. Brachycephaly
c. Hairy (low hairline) forehead
d. Mask like (grim, devoid of expression) facies
e. Hypertelorism
f. Antimongoloid slant of palpebral fissures
g. Synophrys (prominent bushy, confluent eyebrows joining at the nose) (99%)
h. Long and curly eyelashes (99% ),Depressed nasal bridge (83%)
j. Small nose
k. Anteverted nares (88%)
l. Micrognathia (84%)
m. Long bulging, prominent philtrum (94%)
n. Thin upper lip (94%)
o. Central depression below lower lip
p. Down turned corners of the mouth (94%)
q. Late eruption of teeth
r. Widely spaced teeth (86%)
s. High arched palate (86%)
t. Low set, malformed, and hairy ears (70%)
u. Low posterior hairline (92%)
v. Short neck (66%)
w. limbs defect
Slide 9

Hurler syndrome
coarsening of facial features characteristic of this disorder include
 prominence of the forehead
 a flattened nasal bridge
 short broad nose
 widening of the lips
 Features appear puffy due to thickening of the skin.

Enzyme replacement therapy can help temporize treatment, but


the only significant treatment for infantile onset disease is by bone
marrow transplantation, which stabilizes the disorder but does not
improve bony abnormalities.
Slide 10
Slide 11

Menkes disease
characteristic of this disorder include
 loss of milestones
 seizures
 mental retardation
 weakened bones with fractures.
 Low serum copper and ceruloplasmin levels.
Slide 12
What is the likely diagnosis?

Answer
Noonan’s Syndrome

 Widely spaced eyes


 low-set ears
 webbing of the neck
 shield chest, pectus
 increased carrying angle of the arms.
 Common cardiovascular defects include pulmonary stenosis
inassociation with a dysplastic pulmonary valve, atrial septal
defect, and hypertrophic cardiomyopathy.
Slide 13

Answer
• Proptosis , high forehead, mid facial hypoplasia.
• Crouzon’s syndrome.
• Autosomal Dominant
Slide 14
1. Diagnose
2. Inheritance
3. 2 associated abnormalities
4. Investigation for Raised ICT

Answer
1. Apert syndrome
2. Autosomal dominant
3. Proptosis, syndactyly, MR
4. 3 D CTScan
craniosynostosis
Localised
Coronal suture only
 wide skull ( brachycephaly)
Sagittal suture only
 a long narrow skull ( schaphocephaly)
Plagiocephaly - flattening of skull
 single coronal parital
 Single metopic frontal
 Two metopic tringlocephaly
 Single lambdoid occipital /ear displaced posteriorly
 Positional moulding /ear displaced anteriorly.

Generalised
Multiple sutures resulting in microcephaly and developmental delay seen in
 with syndactyly in Apert's syndrome (acrocephalosyndyctyl) – AD
 with exophthalmos in Crouzon's syndrome (craniofacial dysotosis) - AD
Dx and Rx

The fused suture may be felt or seen as a palpable ridge and confirmed
on skull X-ray or cranial CT scan.
If necessary,the condition can be treated surgically because of raised
intracranial pressure or for cosmetic reasons.
Such operations are performed in specialist centres for craniofacial
reconstructive surgery.

Management of craniosynostosis is directed at preserving normal skull


shape and consists of excising the fused suture and applying material
to the edge of the craniectomy to prevent reossification of the bone
edges. The best cosmetic effect on the skull is achieved when surgery
is done during the first 6 months of life
Apert syndrom :AD
acrocephaly ,midfacial hypoplasia ,symetrical syndyctaly
craniocyanostosis (cornal ,sagital ,lambloid)
Crouazon syndrom :AD
acrocephaly ,midfacial hypoplasia ,hypertolerism
craniocyanostosis(cornal ,sagital ,metopic)
Slide 15

1. Give the diagnosis


2. Give five important components
3. Name an important metabolic abnormality and its management
Answer
1. Beckwith-Wiedemann Syndrome
2. Important features
 Gigantism (cardinal feature)
 Hypoglycemia seen in ≈50%
 Macrosomia
 Hemihyperplasia or hemihypertrophy
 Macroglossia (cardinal feature)
 Ear-lobe grooves
 Midfacial hypoplasia
 Hepato-splenomegaly
 Omphalocele ,Umbilical hernia (cardinal feature)
 Embryonal tumours
Adult phenotype
 Hemihyperplasia
 Prominent jaw ,Enlarge tongue
 Ear creases and pits
 Enlarged kidneys and other organs
 Normal height
3. Metabolic abnormality & management
 Hyperinsulismic Hypoglycaemia
 Diazoxide / pancreactectomy
Beckwith-Wiedemann Syndrome/maternal uniparental disomy
Sporadic 85% and Paternally derived 11p15.5 duplications
Patients with the Beckwith-Wiedemann syndrome may acquire tumors,
including
 Wilms tumor
 Hepatoblastoma
 adrenal carcinoma
 gonadoblastoma
 rhabdomyosarcoma.
hyperinsulinemic hypoglycemia
Insulin concentrations are inappropriately elevated at the time of documented
hypoglycemia;
The insulin (μU/mL): glucose (mg/dL) ratio is commonly >0.4; plasma insulinlike
growth factor binding protein-1 (IGFBP-1), ketones, and FFA levels are low.
Additional clues include the rapid development of fasting hypoglycemia within
4–8 hr of food deprivation compared with other causes of hypoglycemia ,the
need for high rates of exogenous glucose infusion to prevent hypoglycemia,
often at rates >10–15 mg/kg/min; the absence of ketonemia or acidosis; and
elevated C-peptide or proinsulin levels at the time of hypoglycemia.
Criteria for Diagnosing Hyperinsulinism Based on “Critical” Samples
(Drawn at a Time of Fasting Hypoglycemia: Plasma Glucose <50 mg/dL)
1. Hyperinsulinemia (plasma insulin >2 μU/mL)[*]
2. Hypofattyacidemia (plasma free fatty acids <1.5 mmol/L)
3. Hypoketonemia (plasma β-hydroxybutyrate: <2.0 mmol/L)
4. Inappropriate glycemic response to glucagon, 1 mg IV (glucose >40 mg/dL)
* Depends on sensitivity of insulin assay.
In some patients who have been managed medically, hyperinsulinemia and
hypoglycemia regress over months.
If hypoglycemia 1st manifests between 3 and 6 mo of age or later, a
therapeutic trial using medical approaches can be attempted up to 4 wk with
 frequent feedings
 diazoxide
 somatostatin
 long-acting somatostatin analog octreotide.
 pancreatectomy may be necessary if failure to maintain euglycemia
without undesirable side effects from the drugs
Slide 16
Beckwith-Wiedemann Syndrome/maternal uniparental disomy
CLINICAL FEATURES

 Gigantism (cardinal feature)


 Hemihyperplasia or hemihypertrophy
 Macroglossia (cardinal feature)
 Ear-lobe grooves
 Nevus flammeus
 Midfacial hypoplasia
 Prominent occiput
 Omphalocele ,Umbilical hernia (cardinal feature)
Adult phenotype

 Hemihyperplasia
 Prominent jaw
 Enlarge tongue
 Ear creases and pits
 Enlarged kidneys and other abdominal organs
 Normal height
Slide 17
Slide 18
Silver-Russell Syndrome/Maternal uniparental disomy
Sporadic occurrence in majority of cases ,Maternal uniparental disomy (UPD) for
chromosome 7 about 10%

CLINICAL FEATURES

Relative macrocephaly ,Large anterior fontanelle (delayed closure)

 Small stature of prenatal onset

Small triangular face , Blue sclera

Downward slanting corner of the mouth

Irregular spacing of the teeth

Ear anomalies

Limb hemihypertrophy

fifth fingers clinodactyly

Bilateral camptodactyly ,Syndactyly

Transverse palmar crease

Café-au-lait spots

Syndactyly of the toes

Normal intelligence
Slide 19
1. Give the diagnosis ?

2. Give four important components ?

Answer
1. Seckel Syndrome

2. Important features

a. Microcephaly
b. Beak like nose “Bird face”
c. Hypertelorism
d. GU anomalies
e. Growth retardation
Slide 20
18 month old child with pallor and cutaneous markers at the back with an
abnormal skeletal profile. P/S was consistent with macrocytic anemia and
pancytopenia

1. What is the most probable diagnosis?


2. Name the cutaneous marker.
3. What malignancies are expected in this patient?

Answer:
1. Fanconi anemia
2. Café au lait spot
3. Acute leukemia
Slide 21
This child has epilepsy, MR, ataxia, unprovoked laughter, severe speech delay

1. Diagnosis?
2. Chromosome involved
3. Inheritance
4. Name a syndrome with the same chromosomal abnormality

Answer

1. Angelman syndrome
2. 15q11.2-12 Uniparental disomy - usually maternal (60 75%)
- occ. Paternal
- normal chromosome
3. ? Dominant mutations
4. Prader Willi - 15q11-13 (paternal origin)
Angelman syndrome/maternal uniparental disomy
caused by maternal deletion of chromosome 15q11-13 in 70% or Imprinting and
paternal uniparental disomy (UPD15) (30% ) the affected child has no maternal
(but two paternal) copies of chromosome 15q 11-13

this can be detected by DNA analysis

CLINICAL FEATURES

 Flat occiput + Occipital groove


 Light hair and eye color
 Deeply set eyes + Strabismus
 Wide open mouth, Macrostomia , Tongue thrusting (happy disposition)
 Widely spaced teeth , Prominent lower lip ,Frequent drooling
 Excessive chewing/mouthing ,hand flapping behaviors
 Prognathia, Pointed chin
 Hypopigmented skin
 Scoliosis and joint contractures increasing with age
 pt need psychological and education support ,AED if has eplepsy

children with Angelman syndrome showing happy disposition,


an open mouth expression, widely spaced teeth, and a pronounced mandible.
Slide 22
This child came with neuroregression from 7 months of age with abn hand
movts

1. Diagnosis / DD
2. 3 salient features
3. Which milestones are delayed?

Answer

1. Retts syndrome / Autism


2. Autism, stereotypies,
microcephaly, loss of
fine purposeful hand movements
3. Social, language
Slide 23

Prune- Belly Syndrome


Important components
 Abnormal abdominal musculature
 cryptorchidism
 Renal and ureteric anomalies with VUR
 Pulmonary hypoplasia

Long term complication is ESRD


Prognostic factors
 Degree of renal dysplasia
 Pulmonary hypoplasia`
Slide 24
An anxious mother of a day 5 neonate has brought her baby with history of
following rash since birth.

1. What is this condition


2. What is the etiology
3. How complications you can encounter
4. What is the management

Answer

1. Klippel Trenaunay weber syndrome


2. cute nous cavernous vascular malformation
3. Cellulites, Thrombophelitits, Heart failure, Gangrne
4. corrective surgery and supportive care
Slide 25
Slide 26
Slide 27

Dandy-walker syndrome
cystic expansion of the 4th ventricle in the posterior fossa and midline cerebellar
hypoplasia, which results from a developmental failure of the roof of the 4th ventricle
during embryogenesis .
Approximately 90% of patients have hydrocephalus
Slide 28
This male neonate was born with multiple fractures and deformities of the
limbs

1. Identify the condition

2. How is the condition inherited?

3. What is the biochemical defect?

4. What are the medical treatment options of this condition?

Answers

1. Osteogenesis Imperfecta

2. Autosomal dominant

3. Reduction in collagen formation

4. a. Growth hormone

b. Bisphosphonates/ allendronate/pamidronate
Slide 29
A 7 month old boy presents with pallor , tri-phalangeal thumbs and mild
hepato-splenomegaly.Hb- 7g%, MCV- 100 fl, Reti.count- < 1%, P.smear-
normocytic to macrocytic RBCS, normal WBCs and platelets.Vitamin B12 and
folate levels are normal.
Hb electrophoresis- raised HbF.
1. What is the diagnosis?
2. What is the underlying defect?
3. What is the closest Differential diagnosis ?
4. Give two points to differentiate them .
5. Name at least one malignancy it can predispose to.

Answers
1. Diamond Blackfan syndrome.
2. Decrease in number and function of erythroid precursors with probable
insensitivity to EPO
3. TEC (Transient Erythroblastopenia of childhood)
4. Age of onset(TEC usually beyond 6 months), MCV(normocytic in TEC),
h/o preceding viral infection( present in TEC), HbF& ADA( Both
increased in DBA)
5. AML, MDS, Osteosarcomas
Slide 30

1. What is the likely diagnosis?


2. What is the mode of inheritance?
3. State True or False
a. Limb shortening is greatest in proximal segments.
b. Fingers often have a trident configuration.
c. Lumbar canal stenosis generally does not develop till early adulthood.
Answer

1. Achondroplasia
2. Autosomal dominant
3. True/ false
a) True
b) True
c) True
Slide 31
Slide 32
12 yrs old male child is brought with history of poor growth
1. Write two obvious abnormal physical findings
2. What is the probable diagnosis?
3. Give one differential diagnosis
4. What is the inheritance pattern of this condition and write 4 diseases with similar
inheritance?
5. Mention 4 abnormal radiological findings in these children
6. Write 2 neurological complications seen in these children

Answer
1) Disproportionate short stature Proximal shortening Bowing of legs
2) Achondroplasia
3) Hypochondroplasia
4) Autosomal dominant
Apert syndrome
Crouzon syndrome
Marfans syndrome
Neurofibromatosis
Osteogenesis imperfect
5) Short tubular bones /Short vertebral pedicles through out the spine /interpedicular
distance decreases /Iliac bones short and round with flat acetabular root /Calvarial bones
are large
6) Hydrocephalus
Spinal cord compression at foramen magnum and lumbar spine
Slide 33
Slide 34

Answer

 ambiguous genitalia family counseling /CAH/AR/25%


 Dexamethasone given to the mother around the time of
conception, and continued if the fetus is found to be female,
Slide 35
Slide 36
1. Identify the clinical condition ?
2. What are the physical findings?
3. What is the metabolic abnormality?

Answer

1. Beckwith Wiedemann syndrome.


2. Hemihypertrophy,omphalocoele, macrosomia, visceromegaly
3. Hyperinsulinemic hypoglycemia.
Slide 37

Due to failur closure of anterior abdominal wall ,the bowel protrudes through a defect in
the abdominal wall, adjacent to the umbilicus 3 cm to right, and there is no covering sac
It is not associated with other congenital abnormalities

Treatment

NG tube for decompression to empty stomach ,the abdomen content wrapped in several
layers of clingfilm or Place in bowel ,bag or wrap in warm saline soaked gauze and saran
wrap ,an intravenous infusion of dextrose established. Many lesions can be repaired by
primary closure of the abdomen.
Slide 38
This female neonate was born with a large mass relation to the umbilical cord

1. Identify the condition?


2. Give three important aspects that you will take care of in the transport of
such a neonate.?

Answers

1. Exomphalos major/ omphalocele

2. important aspects in the transport

a. Transport supine with the hernia suspended by a string


b. Cover the omphalocele with a water proof covering
c. Provide additional fluids
Slide 39

1. What is the diagnosis


2. What is the embryologic basis of these defects?
3. Two other congenital defects associated with this condition
4. Name four teratogenic drugs producing this defect
5. Ideal age for correcting this malformation

Answer

1. Cleft lip and palate


2. Cleft lip ,Hypoplasia of mesenchymal layer resulting in failure of
medial nasal and maxillary process to join
Cleft palate ,Failure of palatal shelves to fuse

3. Congenital heart diseases, hypoplasia or agenesis of thymus and

parathyroid, hypoplasia of auricle.

4. Phenytoin, carbamazepine, prednisolone and alcohol.


5. Lip - 3 months
Palate - 12 months
Slide 40
1. Give the diagnosis.

2. Give three important complications.

3. List three treatment modalities

Answer

1. Giant haemangioma

2. DIC ,Haemorrhage ,Ulceration ,Infection

3. Watchful waiting

Steroids

Interferons

Excision
Slide 41
1. What is the abnormality seen?
2. What is the diagnosis?
3. What other investigation would you advise in this case?

Answer
1. Absence of depression of the right angle of mouth while crying
2. DAOM (deficiency of depressor anguli oris muscle) 2D echocardiography as
the condition is associated with cardiac
3. anomalies
Slide 42
Both of these babies are 1 day old.

The baby in (i) was born vaginally.The father of this baby has a similar
problem.

The baby in (ii) was born by ELSCS with forceps assistance.

(a) What are the diagnoses?

1. Left lower motor neurone 7th nerve palsy

2. Bilateral 7th nerve palsy

3. Right lower motor neurone 7th nerve palsy

4. Right upper motor neurone 7th nerve palsy

5. Left upper motor neurone 7th nerve palsy

(b) Can any further test be performed?

1. Cranial MRI scan

2. Cranial ultrasound

3. 7th nerve conduction study

4. Electromyography

5. None
Answers:

1.(a) (i) 5, (ii) 3

2.(b) 5

Facial nerve palsy

This is usually congenital, but can be acquired as a result of trauma.


The congenital form can involve just the 7th nerve and shows the usual upper
motor neurone signs. It may be associated with others such as bilateral 6th
nerve palsy and bilateral 7th nerve palsy (Möbius syndrome).

If traumatic in orgin,it is usually of the lower motor neurone type; following


forceps delivery, it is very rare, and recovery can be within a couple of
weeks.The congenital form will not disappear, and cosmetic surgery and
looking after the eyes are very important.
Slide 43
Slide 44
In the following picture :

1. whats the most likely diagnosis?


2. what do you expect to find in his lower extremities?

Answer
1. myelomeningocele.
2. hypotonia
Slide 45
Slide 46
Slide 47

1. What is the condition?


2. What is the root value of nerves involved?
3. What is the life threatening complication associated

Answer

1. Erb- Duchenne palsy


2. C5,C6
3. Diaphragmatic paralysis
Slide 48
in the following image:
1. what is the name of this sign?
2. what is the next examination?

Answer

1. sunset eyes (setting sun sign).


2. Head circumference.
Slide 49
1. Diagnosis
2. Commonest differential
3. Management

Answer

1. Epulis: granular cell tumor of gum


2. Hemangioma
3. Surgical excision ,Carbondioxide laser excision
Slide 50
Slide 51
Slide 52

1. Describe the findings in this photograph ?


2. What is the nutritional status of this child ?
3. What are the causes of abdominal distention in this child ?
4. What are the life threatening emergencies associated with this condition ?

Answer

2. Generalized wasting, no edema, alert


3. Marasmus
4. Worm infestation, hypokalemia, T.B. peritonitis or disseminated tuberculosis
5.
 Hypoglycemia
 Hypokalemia
 Hypothermia
 Fulminant sepsis
Slide 53

What the most prominent feature in this child?

Muscle wasting
Slide 54
Slide 55
Slide 56

A 1 month old child was brought with pink staining of the diapers since birth.
He also had history of blisters on sun exposed areas.
On examination the color of the urine is as shown.
1. What is your diagnosis?
2. Name the deficient enzyme.
3. What are the other clinical manifestations?
4. Mention the diagnostic lab finding.

Answer:
1.Congenital Erythropoietic Porphyria (Gunther disease)
2.Uroporphyrinogen III synthase (UROS)
3.Bullous cutaneous photosensitivity- hydroa aestivale, hypertrichosis,
erythrodontia, hemolytic anemia, bone abnormalities
4.Markedly elevated levels of uroporphyrin I and coproporphyrin I in
urine,stool and RBC's.
Slide 57
6 yr old boy brought with the deformity shown in the figure. His sister, 8 yrs
also has a similar deformity gradually increasing since 3 yrs of age. On
examination his height is below the third percentile & US/LS ratio is 1.6:1.
There are multiple dental abscesses. Mother also has a similar deformity but
very mild in nature. S. calcium : 8.8 mg/dl, Phosphorus : 1.8 mg/dl Alk
phosphatase: 380 IU PTH levels: 18 pg/ml (9-55 pg/ml) 25-OH Cholecalciferol:
26 ng/ml(10-50 ng/ml)

1,25(OH)2Cholecalciferol: 18 pg/ml (20-60 pg/ml)

TMPO4/GFR: reduced. X-ray of both

lower limb shows florid richetic changes

1. What is the Diagnosis?


2. What is the inheritance ?
3. It is due to mutation of which gene?
4. What is the Treatment?
5. Write 4 endocrinal causes of short stature.
Answer

1. Hypophosphatemic Rickets
2. X-linked dominant
3. Mutation in PHEX gene
4. Treatment: Phosphorus supplement orally- 1-4 g/day QDS Alfacalcidol- 25-
50 ng/kg/day or calcitriol 0.25-2 mcg/day.
5. 4 causes-
a. 1.Pan hypopitutarisum 2.Isolated GHD
b. Hypothyrodisum
c. 4. Cushing syndrome
d. DM
Slide 58
Slide 59
Slide 60
Slide 61
An eight year old child presented with malaise, anorexia, vomiting, muscle
weakness and orthostatic hypotension. He had H/o a febrile illness with
purpuric rash a few days before.

1. What is your diagnosis?


2. What is the significant finding in the picture?
3. What is the cause of this finding?
4. What is the possible infection preceding it and what is it called
5. What are the possible electrolyte abnormalities ?
6. What is the definitive test for diagnosis of this condition.

Answer
1. Adrenal failur
2. Hyperpigmentation of the gingival and buccal mucosa.
3. Cortisol deficiency leading to increased ACTH production and Melanocyte
stimulating hormone arising from the ACTH precursor POMC.
4. Meningococcemia – Waterhouse-Friderichsen syndrome
5. Hypoglycemia, ketosis, hyponatremia
6. ACTH Stimulation test
EYES
Slide 1
Slide 2

1. Dx?
2. Causes ?
3. Rx ?
Answer
1. Bitot’s spots
2. Vit A deficiency
3. Vit A 6 mo to 1 year 100000 IU oral repeated after 48 hrs
1 yr to 6 years 200000 IU oral repeated after 48 hrs
Slide 3

CHARGE ASSOCIATION
Sporadic in most cases

•C for Coloboma (80–90%) of the iris


•H for Heart disease (cardiovascular malformations) (85%)
•A for Choanal Atresia (50–60%)
•R for Renal anomaly (70–80)
•G for Growth and mental Retardation (70–100%)
•E for Ear anomalies/deafness (90%)

1. Major criteria

a. Eye anomalies
b. Choanal Atresia (50–60%)
c. Ear anomalies/deafness (90%)
d. Cranial nerve dysfunction (70–90%)

2. Minor criteria

a. Heart disease (cardiovascular malformations) (75–85%)


b. Growth and mental Retardation (70–100%)
c. Genital anomaly (70–80%)
d. Orofacial cleft
e. Tracheoesophageal fistula/esophageal atresia (15–20%)
f. Distinctive face (70–80%)

Need Audiology, tympanometry, fundoscopic ,NG feeding tube ,Echocardiography and


Abdominal Ultrasound,CT scan and/or MRI for brain and genetic analysis.
Slide 4

1. Give the diagnosis.


2. State mode of inheritance.
3. Name five important components.
4. Name most frequent immunological defect.
Answer
1. Ataxia – Telengiectasia
2. AR
3. Important components
 Cerebellar ataxia
 Oculo-cutaneous telengiectasia
 Immuno-deficiency
 Sino-pulmonary infections
 Lympho - reticular malignancies

4. Selective absence of Ig A
Slide 5
Slide 6

1. What is the clinical condition called?


2. Write common causes of leukocorea.
3. Which is the commonest malignancy associated with this?
4. Which chromosome is implicated in malignancy mentioned in question
5. What other malignant disorders are associated with this malignancy?

Answers

1. Leucocoria
2. Cataract
retinoblastoma
Pupillary membrane
Vitreous opacity
Retrolental fibroplasias
Retinal detachment

3. Retinoblastoma
4. RB1 gene Chr 13q 14
5. Osteosarcoma
Soft tissue sarcomas
Malignant melanoma
Slide 7

This 5 yr old comes with unsteadiness of gait with recurrent respiratory


infections.

1. Diagnose the condition


2. Inheritance
3. 3 salient features
4. 2 lab
5. investigations

1. Ataxia Telangiectasia
2. Autosomal recessive
3. Ataxia, telangiectasia, immunodeficiency,recurrent sinopulmonary
infections, X ray hypersensitivity, malignancies
4. Alpha feto proteins, Immunoglobulins,(lowIgA, IgG, high IgM),
chromosomal breaks with exposure to radiation.
Slide 8
Slide 9
Slide 10

Fundus Signs of Papilledema


Slide 11

1) What is the diagnosis


2) Mention 5 diseases with similar findings
3) Earliest clinical presentation of this condition
4) Name two treatable conditions with the same findings

Answer
1) Retinitis pigmentosa
2)
A. MPS
B. Lawrence moon biedl syndrome
C. Refsums disease
D. Abetalipoprotenimia
E. Ushers syndrome

3) Night blindness

4) Refsums disease and abetalipoprotenemia


Slide 12

1. What is your diagnosis?


2. Name 4 conditions in which it is seen.
3. What is the underlying mechanism?

Answer

1. Cherry Red spot


2. Tay sachs disease, Sandhoff disease, Niemann pick disease, central
retinal artery occlusion.
3. Due to loss of transparency of the retinal ganglion cell layer secondary to
lipid accumulation/edema. Because ganglion cells are not present in the
fovea, the fovea transmits the underlying choroidal colour.
Slide 13
Slide 14

NEUROFIBROMATOSIS TYPE 1 (NF1)

It is an autosomal dominant

In order to make the diagnosis, two or more of these criteria need to be present:

· six or more café-au-lait spots >5 mm in size before puberty, >15 mm after puberty

· more than one neurofibroma (firm nodular overgrowth of any nerve)

· axillary freckles

· optic glioma which may cause visual impairment

· one Lisch nodule (hamartoma of the iris seen on slit-lamp examination)

· bony lesions from sphenoid dysplasia which can cause eye protrusion

· first-degree relative with NF1.

Also there Megalencephaly,learning difficulties and epilepsy .

Need Audiology, tympanometry, fundoscopic , and ,CT scan and/or MRI for brain show
hyperintense area in B.ganglion and internal capsule and genetic analysis.
Slide 15

A 11 yr old male child is admitted with sudden change of behavior, slurring of


speech, ataxia and dystonia. On investigations, he found to have anemia with
renal tubular acidosis. His ophthalmic examination was done and shown in fig.

1. Describe the findings?


2. What is the most possible diagnosis
3. What is the pattern of inheritance for it?
4. What are the most specific investigations?
5. What is the complete treatment in this case?
Answer

1. Slit lamp examination showing brown discoloration at outer margin of


cornea
2. Wilson disease with lenticular degeneration
3. Autosomal recessive
4. Hepatic copper content (µg/gm dry wt. of liver- it exceeds >250 µg/gm
dry wt.)
5. D-Penicillamine with Pyridoxine and Zinc and all family members should
be screened with s. ceruloplasmin and urinary excretion of cu, slit lamp
examination
Slide 16
The parents of a 2-month-infant who you are seeing for a health supervision
visit relate that a red mass on the infant’s eyelid has grown and no longer is
the flat birthmark seen in the newborn nursery. Physical examination reveals
a hemangioma located on the upper lid. The visual axis is not obstructed, but
the infant is unable to open her eyelid completely. The remainder of the
examination findings is normal, with no evidence of other hemangiomas.

1- What is the most important reason for urgent pediatric ophthalmologic


referral for this infant?

2-What 2 interventional procedures should be advised when there is evidence


of this ophthalmologic complication?

3- What lab. Investigation should be ordered before the procedures?

Answer

1- The infant described is at risk for amblyopia or diminished visual acuity


because of potential obstruction of vision by a hemangioma. Thus, urgent
ophthalmologic referral is indicated.
2- If ophthalmologic evaluation reveals evidence of amblyopia, laser therapy
or intralesional corticosteroids may be used to reduce the size of the
hemangioma.
3- Platelets count to exclude Kasabach-Merritt syndrome.
Slide 17
SKIN LESSION
1. Colloiden baby ,harlequin ichthyosis ,Dx and mode of inheritance
2. erythemia margnatium ,rheumatic fever: how to-diagnose it
3. Seborrheic dermatitis.
4. Tinea versicolor
5. peroneal skin rash , Dx acrodermatitis enterohepatica
6. Cold panniculitis
7. Pityriasis alba
8. Angioedema
9. scleroderma
10. kawasaki ,Dx and Rx
11. Facial hemangioma ,SWS ,complication
12. Café au lauit Dx ,complication, other finding
13. shagreen patch (TS)
Slide 1
Slide 2

 Harlequin baby syndrome


 AR

This is recessively inherited, and various abnormalities of keratinization and epidermal


lipid metabolism have been demonstrated. Usually the baby is covered with thick dark
plates of scale with severe ectropian, deformed ears, and claw hands and feet.
Most babies die as neonates and those who survive will develop a type of ichthyosis.
Slide 3
Slide 4

This is a collodion baby.

The prognosis is uncertain with only a few babies having no sequelae.

The majority manifest feaures of lamellar icthyosis or congenital icthyosiform


erythroderma in later life (both forms of icthyosis).
Slide 5

The disorders can be categorized under three major headings with multiple

subgroupings:

 epidermolysis bullosa simplex (EBS)


 junctional epidermolysis bullosa (JEB)
 dystrophic epidermolysis bullosa (DEB)
Slide 6
Slide 7
4 yr old child with history of chronic diarrhea was brought with complaints of
perioral and perineal rash.

• What is your diagnosis?

• Mode of inheritance?

• Investigation used for diagnosis?

• Treatment of choice?

• Mention a few differential diagnosis.

ANSWER

• Acrodermatitis enteropathica.

• Autosomal recessive - defect in intestinal zinc specific transporter gene.

• Low plasma zinc concentration.

• Zinc - 50mg/day for infants, upto 150mg/day for children.

• MSUD, organic acidemia, methylmalonic acidemia, biotidinase def, PEM.


Slide 8
1. What is the diagnosis? (2)
2. What is the mode of inheritance? (2)
3. What are the 4 stages? (4)
4. Name 2 associated defects.(2)

Answer

1. Incontinentia pigmenti/ Bloch- Sulzberger disease.


2. X-Linked dominant
3. A) Vesicular streaks B) Hypekeratotic plaques
C) Pigmentary stage D) Hypopigmentation

4. Associated Defects:
 Dental: Delayed dentition, conical teeth, impaction
 Skin: Alopecia, Nail dystrophy,
 CNS: Dev.delay, microcephaly, spasticity, seizures
 Ocular: Microphthalmos, optic atrophy, strabismus, cataracts,
retrolenticular masses, neo-vascularisation
 Skeletal defects
Slide 9
You are evaluating a term infant in the neonatal intensive care unit for a
seizure that occurred 1 day after birth. Her neonatal course has been
otherwise unremarkable. The only significant finding on physical examination
is an erythematous patch involving the left side of the face, including the
upper and lower eyelids as shown below

1-What is your diagnosis?


2-What is the most common ophthalmologic finding associated
with this condition?
3-What other features may be seen in this infant?

Answer

1- Sturge-Weber syndrome.
2- The most common ophthalmologic finding associated with this condition is
glaucoma.
3- Affected infants may have seizures, and some children have developmental
delay, learning disabilities, and intellectual disability.
Slide 10

1. Diagnosis
2. 2 associated abnormalities
3. Classical CNS picture in CT scan and is described as
4. Inheritance

Answer

1. Sturge Weber syndrome


2. Glaucoma, intracranial calcification, intractable epilepsy, hemangioma
3. Tram track calcification
4. Sporadic inheritance
Slide 11

Answers

1. Sturge weber syndrome

2. Sporadic

3. Tram ,railroad track appearance and hypoperfusion and calcification by CT

4. Bupthalmos (glaucoma)
Slide 12

The cardinal manifestations of Sturge-Weber syndrome are as follows:


1. A vascular malformation or port-wine stain over the face that involves the
cutaneous distribution of the ophthalmic division of the trigeminal nerve
2. Ipsilateral leptomeningeal angiomatosis with associated intracranial
calcifications
3. A high incidence of mental retardation and ipsilateral ocular Complications
Slide 13

1. What is the condition?


2. Describe the lesions.
3. What is the mode of inheritance?
4. What is CNS association?

Answer

1. Tuberous sclerosis( ch 9,16)


2. depigmented 'ash leaf'-shaped patches which fluoresce under ultraviolet
light , roughened shagreen patch, angiofibroma (adenoma sebaceum)
3. Autosomal dominant
4. Characteristic brain lesions are tubers located in convolutions of
cerebral hemisphere typically in the subependymal region with MRI.

Need EEG (epilepsy , infantile spasms), fundoscopic (phakomata) ,


echocardiography (rhabdomyomata), abdominal US(PKD), and ,CT scan
Slide 14
You are evaluating a 15-month-old child for mild developmental delay. He sits
well alone, crawls, claps his hands, and waves, but he does not pull to stand or
say any words. His father and paternal uncle have epilepsy and attended
special education classes when they were in school. You note three
hypopigmented macules scattered over his trunk and arms.

1-What is the most likely diagnosis?


2- Name this skin lesion?
3-What is the significance of this skin lesion?
4-What is the most appropriate next step in this boy’s evaluation?
5-What additional diagnostic testing should be done?

Answer
1- Tuberous sclerosis complex (TSC).
2- Hypomelanotic macule, sometimes called ash leaf spot or Fitzpatrick patch.
3- The presence of three or more hypomelanotic macules is a major criterion
for the diagnosis of TSC.
4- Cranial magnetic resonance imaging is necessary to determine the presence
of brain hamartomas.
5- Additional diagnostic testing should include ophthalmologic evaluation,
neurodevelopmental testing, echocardiography, and renal ultrasonography.
Although the boy’s delayed speech suggests the need for audiology
evaluation.
Slide 15
Slide 16

Tuberous sclerosis
This child has tuberous sclerosis. This is inherited as an autosomal dominant
condition with an estimated frequency of 1:6000, although half of the cases
are sporadic. The disease is heterogeneous with a wide spectrum of clinical
presentation ranging from severe mental retardation and frequent seizures to
normal intelligence and lack of seizures. Characteristic‘tubers’ may be located
in the subependymal regions of the brain and may undergo calcification. Other

anomalies may also be found in the heart (rhabdomyoma), kidneys


(hamartoma), eyes (optic nerve mulberry tumour and/or retinal phakoma),
lungs (angiomyolipoma) and bone. Skin abnormalities include hypo- or
hyperpigmented areas, sebaceous adenomas over the cheeks and nose, and
shagreen patches typically over the lumbar region.

Subungual fibromas may be found around toenails and fingernails.


Hypopigmented areas are best identified usingWoods

light (UV light source). CT and MRI brain scans identify cerebral anomalies.

EEG may demonstrate hypsarrhythmia, particularly where presentation is in

infancy in the context of infantile spasms. Echocardiogram will identify


cardiac lesions.
Slide 17
1. Name 2 other skin lesions
2. Inheritance
3. Other systems you would examine
4. Name 3 lesions seen in the brain

Answer

1. Ash leaf macule in Tuberous sclerosis


Adenoma sebaceum , shagreen patch, cafe au lait spots
2. Autosomal dominant
3. Eyes, heart, kidneys, brainTubers, subependymal nodules,
4. subependymal giant cell astroc
Slide `18
Slide 19

1. Name the neurocutaneous marker seen

2. In NF-1 what is the diagnostic criteria with respect to this marker?

3. Name a X-linked dominant neurocutaneous syndrome

4. What are the three stages of syndrome in question 3?

Answers
1. Café-au lait macule

2. Six or more CAL macules larger than 5 mm in greatest diameter in prepubertal And
larger than 15 mm in greatest diameter in postpubertal individuals

3. Incontinentia pigmenti

4. Stage 1-Vesicular stage

Stage 2-Verrucous stage

Stage 3-Pigmented stage


Slide 20

1) What is the diagnosis?

2) What 2 findings are characteristics?

3) What is the effect of treatment?

4) List 4 drugs useful

5) What physical therapy will help?

Answer

1) Psoriasis

2) a) Plaques with yellowish white scale like mica

b) Auspitz sign – pinpoint bleed or removal

3) Koebner phenomenon lesions appear

4) Coal tar, topical steroid, salicylic acid, calcipotriene (Vit.D analog),

Methotrexate & cyclosporine and retinoid

5) UV light
Slide 21

1. Name the condition?


2. What is the natural course of these lesions?

Answer

1. Strawberry angiomas
2. Lesions increase over the first few months of life and then regress
Slide 22
Slide 23
Slide 24
Slide 25
Slide 26
A 3-year-old boy is brought to the clinic by his parents due to concerns about how
easily he bruises. They say that since he began walking at 18 months, he
frequently has large, purple bruises that appear with no known history of trauma.
They do not believe that he falls more frequently than other children his age, and
they deny a family history of easy bruising. On physical examination, the normally
grown child has prominent eyes, a delicate and narrow nose, and numerous
bruises in various stages of healing, primarily overlying his shins but also
scattered elsewhere on his body. He has translucent skin over the chest, with
prominent vascular markings, and his fingers are slender and hypermobile.

1-Which condition is most consistent with this boy’s features?


2-What is the most common type of inheritance?
3-What advice you should give to the patient and his parents when they counsel
you?

Answer
1- Ehlers-Danlos syndrome.
2- Most types of EDS are autosomal dominant, but autosomal recessive and
Xlinked forms also are described.
3- My advice is to avoid activities that can place affected individuals at increased
risk for arterial rupture, such as collision sports, weight training, and heavy lifting.
Slide 27

This child also has joint hypermobility

1) What is the diagnosis?


2) What is the usual mode of inheritance?
3) What is the defect?
4) How many clinical forms?
5) What cardiac among can occur?
6) What surgical emergencies?
7) Difference with Cutis Laxa

Answer
1) Ehlers Danlos
2) AD
3) Defect of fibrillar collagen – quantitative
4) 10
5) MVP, AR
6) Rupture of great vessels, dissecting aneurysm, stroke
7) Cutis Laxa – skin hangs in redundant folds – AR
EDS – hyperextensible snaps back into place when stretched - AD
Cutis Laxa – Bloodhound appearance, aged appearance
Hyperelasticity and hypermobility of joints, hoarse cry, lax vocal cords.
Slide 28

Cutis Laxa – skin hangs in redundant folds – AR

Bloodhound appearance, aged appearance

Hyperelasticity and hypermobility of joints, hoarse cry, lax vocal cords.


Slide 29

Answer

1. Jenuwy lesion
2. causes
 penicillin and gentamicin
 infective endocarditis
Slide 30
Slide 31
Slide 32
These lesions are tender

1) What is the diagnosis?

2) What 2 common infections and drugs can trigger this?

3) What 2 non - infectious systemic disorders can trigger this?

Answer

1) Erythema Nodosum

2) Sulfa, Phenytoin, Oral contraceptives

3) TB, Streptococcus ,IBD, Spondylo arthropathy, Sarcoidosis


Slide 33
A previously well 6-year-old boy presents with a rash over his lower extremities. He has
had knee and ankle pain for 3 days that has caused difficulty ambulating since this
morning. On physical examination, he is afebrile, and his heart rate is 80 beats/min,
respiratory rate is 16 breaths/min, and blood pressure is 108/60 mm Hg. You note
bilateral ankle swelling and a purpuric rash over his lower extremities.
Laboratory findings include:
 White blood cell count,10x103/mcL
 Hemoglobin, 11.5 g/dL
 Hematocrit, 35%
 Platelet count, 410x103/mcL
 Urinalysis shows:
 Specific gravity, 1.025
 pH, 6
 3+ blood
 Trace protein
 20 to 50 red blood cells/high-power field
 The random urine protein-to-creatinine ratio is 0.15

1. Name probable diagnoses ?


2. what abdomen complication has developed ?
3. Medical treatment (specific for disease – other than blood / resuscitative fluid) ?
4. mention 2 other clinical findings in patient?
Answers
1- The patient does not require renal biopsy at this point because the urinalysis does not
yet show evidence of significant proteinuria.
2- Corticosteroid therapy is not indicated for joint symptoms or purpura.
3- Ibuprofen should be used with caution in children at risk of renal disease, and they
should be avoided in this patient, who already has microscopic hematuria, to prevent
further progression of early renal involvement.
Slide 34
15 year old male presented with pain in abdomen for past 25 days (acute intermittent,
periumblical),also developed swelling over scrotum 6 days back which subsided within 24
hours. Past 2 days patient is having pain over Rt wrist with swelling of Rt knee. Patient
also developed rash over gluteal region. Patient was passing red stool. Per abd
examination showed empty Rt lower quadrant

1. Name probable diagnoses ?


2. what abdomen complication has developed ?
3. Medical treatment (specific for disease – other than blood / resuscitative fluid) ?
4. mention 2 other clinical findings in patient?

Answers
1. HS Purpura
2. Intussusception
3. Steroids
4. Arthralgia , Abdominal pain
Slide 35
1) What are the 4 types of lesions?

2) What bacteria cause this?

3) Which drug can induce this?

4) Name 4 drugs used for Rx?

5) What Dietary advice will you give?

Answer

1) Open Comedones – blackhead ,

Closed comedones – whitehead Papules

pustules and nodulocystic lesions

2) Propionibacterium acnes

3) Corticosteroid, androgens, INH, phenobarbital ,Phenytoin, B12 and lithium

4) Benzoyl Peroxide, Tretinoin, Adapalene ,

Topical – Erythromycin and clindamycin Use for 4 to 8 weeks:

Systemic therapy: Tetracycline, Doxy, Minocycline Isotretinoin (nodulocystic)

(teratogenic) Intradermal triamcinolone

5) Normal
Slide 36

1. Give the diagnosis.


2. Name the most important endocrinal feature
3. Name the most important cardiovascular problem
4. Name the mainstay of management

Answer

1. Congenital Lipodystrophy
2. Insulin resistant DM
3. Hypertrophic cardiomyopathy
4. Dietary fat regulation
Slide 37
Slide 38
Slide 39
Slide 40
A 3-year-old girl is brought to your office for re-evaluation of a fever that
began 6 days ago. Her mother tells you that her daughter's temperature has
been as high as 102.2°F (39°C). Her physical examination was unremarkable
when you examined her 3 days ago, but today you note injected sclera;
cracked, red lips; a strawberry appearance of her tongue; and a swollen,
nontender, cervical node.

3-What treatment you should order?


1-What is the most likely diagnosis?
2-What important test should be done at time of diagnosis and why?
Answer
1- Kawasaki disease (KD) is believed to be a multisystem illness characterized
by vasculitis of small- and medium-size blood vessels, including the coronary
arteries.
2- Echocardiography usually is performed at the time of diagnosis to assess for
the presence of a subclinical myocarditis, to evaluate for coronary arteritis,
and to serve as a baseline for future studies.
3- Intravenous gamma globulin during the acute phase and highdose aspirin
therapy (80 to 100 mg/kg per day) is administered until the patient is afebrile
for 48 hours, at which time the dose is decreased to 3 to 5 mg/kg per day for 6
to 8 weeks or until platelet concentrations normalize.
Slide 41
1. Give the diagnosis ?

2. List clinical staging.?

3. What is the most important sequel and it’s prediction?

4. Outline the treatment.?

Answer

1. Kawasaki Disease

2. Clinical Staging

a. Acute febrile phase

b. Sub-acute phase

c. Convalescent phase

3. Complication and prediction

a. Coronary artery aneurism

b. Onset of coronary artery changes within 2 months of onset of illness

4. Treatment

a. IVIG + High dose aspirin


Slide 42
Slide 43

1. Identify the spot with its grade ?

2. Give the grades of clubbing?

ANSWERS

1. Grade 4 or 5 clubbing

2. Grade 1- Fluctuation and softening of the nail bed


Grade 2- Loss of normal angle
Grade 3- Accentuated convexity of the nail
Grade 4- Broadened terminal pulp of the digit
Grade 5- Shiny and glossy changes in the nail and adjacent skin
Slide 44

Mongolian spots.

Mongolian spots are a form of dermal melanocytosis.

Melanocytes, in their transit from the neural crest to epidermis during the
embryonic period, become arrested in the dermis (migrational arrest). This results
in ectopic melanocytes in thedermis.

Natural history

l age 2 years
Slide 45
An 18-year-old boy presents to the emergency department 30 minutes after eating
at a seafood restaurant. He states that approximately 10 minutes into his meal he
developed generalized hives, pruritus, and difficulty breathing. He has a history of
shellfish food allergy, although he had ordered steak and denies eating any crab,
lobster, or shrimp. On physical examination, the patient appears to have labored
breathing, audible wheezing, and diffuse raised erythematous lesions on his trunk
and extremities. His vital signs include a temperature of 98.5°F (37°C), heart rate
of 100 beats/min, respiratory rate of 22 breaths/min, blood pressure of 110/60 mm
Hg, and pulse oximetry of 92% on room air.

1-What is the most likely cause for the patient condition?

2-What is the most appropriate immediate action?

Answer

1- The adolescent described in the vignette most likely is experiencing an adverse


food reaction, specifically anaphylaxis to shellfish.

2- The rapid (<30 min) onset of urticaria and wheezing in a shellfish-allergic


patient who is eating in a seafood restaurant is likely anaphylaxis and warrants
prompt administration of intramuscular epinephrine.
Slide 46

Nevus of Ota.

other forms of dermal melanocytosis).

These are permanent lesions (unlike the majority of Mongolian spots).


Slide 47

Cold panniculitis,.
is an inflammation of the subcutaneous fat after prolonged exposure to cold.

is a self-limiting condition. There is no specific therapy except reassuring


caregivers.

A-Lesion resolves spontaneously in 2 to 3 weeks without scarring

Post inflammatory hyperpigmentation may persist.


Slide 48

pityriasis alba

to be differentiated from vitiligo by Wood’s lamp examination.

Because the disorder is usually asymptomatic and spontaneously resolves in


several months to a few years, treatment is usually unnecessary, although
moisturizers may help reduce surface scaling.
Slide 49

Seborrheic dermatitis.
differential diagnosis of seborrhea includes Langerhans cell histiocytosis (in
which the rash is generalized, in part petechial, and usually associated with
chronic draining ears and hepatosplenomegaly) and tinea corporis (in which
lesions usually are more circumscribed, with an active border and central
clearing). Scalp lesions may be difficult to differentiate from psoriasis.
The dermatitis of seborrhea is usually nonpruritic and mild in nature. Most
cases respond to topical steroids, and many clear spontaneously, although
residual postinflammatory hypopigmentation may persist for weeks or
months thereafter.
Some practitioners find that use of a topical antifungal cream or wash, as well
as a low-potency topical steroid, hastens resolution.
Antiseborrheic shampoos may also be helpful for patients with
scalp involvement.
Slide 50

Acute hemorrhagic edema of infancy.


Slide 51

Localized scleroderma.

two linear lesions of the face and scalp with associated scalp ulceration and
crust.
Slide 52

Angioedema
Slide 53

Lip-licking eczema.

perioral skin is inflamed, scaly, and thickened as a result of repetitive licking of


the lips.
Slide 54
Slide 55
INFECTION
1. Molluscum Contagiosum ,Dx ,What is the organism? Pox virus
2. Rubella
3. vesicular rash, h/o fever +hepatomegaly given (inf .mononuclosis)
4. Slap cheek – Erythema infectiosum/ Parvovirus B19
5. Hand mouth foot disease
6. Adenovirus Infections
7. chicken pox (Dx, name atypical form in HIV,CD4 change)
8. herpetic gingivostomatitis.
9. Eczema herpeticum (EH)
10. roseola
11. Varicella zoster picture ,what complication
12. Gianotti-Crosti syndrome.
13. whitelow herptic lesion
14. erythema multiform skin rash 2 causative m.o
15. Meningococcemia Dx ,your approach? What is the organism? What AB?
16. purpura fulminans.
17. Skin impetigo
18. scarlet fever.
19. staph scalded skin syndrome
20. erysipelas –causes and Rx
21. Icthyma gangernosum
22. Scapies
Slide 1
This is a PBS and photograph of a 10 year old boy who has presented with
fatigue, fever, and malaise of > lweek in duration.

On examination, he has generalized lymphadenopathy, splenomegaly and


hepatomegaly.

1) describe the main features in each picture

2) what is the diagnosis

3) Write 3 complications that may occur

4) What medication is contraindicated

5) what is the treatment

Answer
1.Picture 1 shows tonsillitis with membrane formation, picture 2 shows
atypical mononuclear cells which are enlarged with irregular nuclei and
basophilic pleomorphic cytoplasm
2.Infectious mononucleosis
3.Splenic hemorrhage/ splenic rupture/ airway obstruction causing drooling/
stridor and respiratory distress/ thrombocytopenia/ Coombs positive
hemolytic anemia/ GBS/ Reyes syndrome
4.Ampicillin and amoxycillin
5.Rest and symptomatic treatment / avoidance of contact sports or strenuous
athletic activities during the first 2-3 weeks of illness or while splenomegaly is
present.
Slide 2

infectious Mononucleosis.

Transmission of EBV can occur by intimate oral contact (i.e., kissing), sharing
eating utensils, transfusion, or transplantation.

The incubation period usually ranges from 30 to 50 days, although it is shorter


(14 to 20 days) in patients with transfusion acquired infection.
Slide 3
This is a picture of a 5 year old boy with a characteristic rash which was
preceded by a prodromal phase consisting of low grade fever, headache and
mild URTI. ®

1. Describe the 2 pictures?


2. What is the diagnosis? And the cause?
3. Mention two immune mediated post infectious of the above organism

Answer

•Erythema infectiosum/ Parvovirus B19

•Picture-1- slapped cheek appearance

•Picture-2- lacy reticulated rash on the arm

•Rash and arthropathy are immune mediated, post infectious phenomenon


Slide 4
This child has fever with URI

1) What is the diagnosis?


2) What is the causative organism?
3) Name one serious hematological complication in this disease.
4) Name one orthopedic complication.
5) Name one cause for intrauterine fetal demise.
6) What treatment is recommended for severe hematological complications?

Answer

1) Erythema Infection or fifth disease

2) Parvovirus B 19

3) Transient aplastic crisis

4) Arthropathy

5) Non – immune fetal hydrops

6) Aplastic crisis in patients with sickle cell disease, other hemoglobinopathies,


and other forms of hemolytic anemia need IVIG
Slide 5
an 8 month infant who developed a rash during the declining phase of fever
starting with the cheeks

1. What is the most probable diagnosis?


2. What is the causative organism?
3. Name two situations where infection with this organism may be life
threatening

Answers

1. Erythema infectiosum/ fifth disease/

2. Parvovirus B 19

1. Aplastic crisis in hemolytic anemia

2. Non-immune hydrops fetalis in fetal infection .


Slide 6

1. -What is the diagnosis?


2. -What is the organism?
3. –What is the Predisposing factor?

Answer

1. Molluscum Contagiosum characterized by discrete pearly white papules


that are umbilicated.
2. Pox virus
3. Predisposing conditions for disseminated lesions
 Children with atopic dermatitis
 Human immunodeficiency virus (HIV) infection
 Other immunodeficiency states including leukemia
Slide 7
You are evaluating an otherwise healthy 3-year-old boy who has developed a
cluster of 5 to 10 flesh-colored, pearly papules, some of which have central
umbilication. He has no prior skin conditions.

1. -What is the diagnosis?


2. -What is the etiology?
3. -What is the most appropriate management?
4. -What is the fate of the lesion if untreated?

Answer
1- Molluscum contagiosum.
2- They caused by a DNA pox virus called molluscum contagiosum virus (MCV).
3- Because of its benign nature and ultimate self-resolution, observation is
frequently the best approach for healthy children.
4- Without treatment, individual lesions usually resolve in 6 to 9 months, and
the infection clears within 1 to 4 years. Molluscum contagiosum often behaves
differently in patients who have HIV and resists treatment more than in healthy
children.
Slide 8

Herpes zoster (shingles).


Involvement of the ophthalmic branch of the trigeminal nerve
produces lesions involving the forehead, eyelids, and nose.
Although varicella can be transmitted by patients with herpes
zoster, contagion is generally less of a problem because most
patients have lesions on areas that are covered by clothing and the
oropharynx is not involved in most cases.
Slide 9
Slide 10
1. Picture A

Picture A

a) Outline the treatment for this condition?.


b) List 4 complications of this condition?.
c) Any medications you would advise for his 5 yr old elder sister and his mother who is 2
months pregnant?

Picture B

a) What is the diagnosis?


b) List 3 clinical features of this condition ?
Answer
I. Varicella
a) Symptomatic treatment with antipyeretics / antihistamines / hygiene etc is adviced if
child is healthy and is uncomplicated varicella.
A cyclovir is started within 72 hrs if child is immunosuppressed / on steroids or salicylate
or child has chronic cardiac / pulmonary disorder or complicated varicella.
The routine use of acyclovir in healthy children is recommended by the AAP if it can be
given within 24 hours after the rash first appears in children older than 12 years

b) Secondary bacterial skin infections


Encephalitis / cerebellar ataxia
Pneumonia
Purpura / HUS
Arthritis
Myocarditis / pericarditis
Pancreatitis
c) Varicella vaccine (72-120 hours /3 to 5 days after exposure) is recommended for 5 yrs
old sister postexposure prophylaxis and for healthy people if have not been infected or
vaccinated earlier or without evidence of immunity 12 months or older.
If mother has not been infected or vaccinated earlier. VZIG can be given to the mother to
prevent her from getting chicken pox but she has to be told that it may not prevent the
fetus from being infected or prevent development of total embryopathy.
if pregnant mother infected treated with aciclovir.

II. a) congenital varicella syndrome


b) Shortened / malformed extremities
Zigzag scarring of skin
Neurological defects including dysfunction of anal + urethral sphineters
developmental defect of eye including hormone syndrome and cataracts.
Slide 11

A. Superinfection of this child’s lesions with group A β-streptococci


lead to purpura fulminans.

B Disseminated hemorrhagic varicella.


Slide 12

Primary herpetic gingivostomatitis.

the virus can infect any area of the skin, the lips and fingers or
thumbs (as in herpetic whitlow) are the most common sites of
involvement.

Ocular herpetic infectionserious and needs urgent management.


Slide 13

czema Herpeticum (Kaposi Varicelliform Eruption).

Prompt treatment with intravenous acyclovir is recommended.


A significant risk of secondary bacterial infection also exists.
Slide 14

Eczema herpeticum (EH)


disseminated cutaneous herpes simplex virus (HSV) infection superimposed on a
preexisting skin disorder (e.g., eczema).

The diagnosis of EH is usually made clinically.

Diagnosis can be confirmed by Tzanck smear, HSV culture, or PCR.

Complications include secondary bacterial infection (Streptococcus pyogenes and


Staphylococcus aureus), sepsis, and scarring.
Slide 15

Herpes Zoster- Ramsay Hunt syndrome.


Slide 16

Herpetic whitlow..
cutaneous infection of the terminal phalanx of the fingers or thumb caused by
herpes simplex virus(usually HSV-1 or HSV-2).

Diagnosis is usually made clinically.

A needle aspiration of the lesion for HSV culture and Tzanck smear will confirm
the diagnosis.
Slide 17

Gianotti-Crosti syndrome.

Liver function tests should be done, and if the results are abnormal, serologic
studies for hepatitis B and EBV should be performed.
Slide 18
A young child presented with fever for one day, a discrete, pinkish
red, fine maculopapular eruption, occipital, posterior cervical, and
postauricular nodes enlargement.

1. Name the finding in photo B?


2. What is the most likely diagnosis?
3. What complications may be seen in children?

Answer.
1. Red palatal lesions (Forschheimer spots).
2. Rubella/German measles.
3. Complications are rare in childhood and include arthritis, purpura
with or without thrombocytopenia, and mild encephalitis.
Slide 19
While evaluating a newborn, you note absence of a red reflex. Subsequent
evaluation by an ophthalmologist reveals bilateral cataracts. The infant also
has failed the neonatal hearing screening test, with results suggestive of
severe hearing loss.

1- What is the most likely cause of the findings in this patient?


2-Enumerate 4 other causes for leukocoria?
3-What other findings you may see in this infant?

Answer.
1- Congenital rubella.
2- A number of different conditions are associated with leukocoria, including
neoplastic conditions (eg, retinoblastoma), retinal abnormalities (eg, retinopathy
of prematurity), developmental abnormalities (eg, chorioretinal coloboma),
inflammatory conditions (eg, toxocariasis), and other conditions, such as
cataracts.
3- Common findings in infants who have congenital rubella include eye
abnormalities, such as cataracts, congenital glaucoma, and retinopathy, as well
as sensorineural hearing loss, heart disease (peripheral branch pulmonary artery
stenosis, patent ductus arteriosus), and mental retardation. Affected infants
often also have intrauterine growth retardation, hepatosplenomegaly,
thrombocytopenia, and purpuric skin lesions (blueberry muffin rash).
Slide 20

Adenovirus Infections.
 Conjunctivitis
 upper respiratory tract infections
 pharyngitis
 croup
 bronchitis
 bronchiolitis and pneumonia (occasionally fulminant)
 gastroenteritis
 myocarditis
 nephritis
 cystitis,
 encephalitis.
Slide 21
A 13-year-old boy presents to the emergency department with a 3-day history
of severe sore throat and fever. He is having trouble swallowing due to pain.

1-What is the most likely diagnosis?


2-What are the causative agents?
3-Enumerate 4 differential diagnoses for the mouth lesion?
Answer
1- Hand-foot-and-mouth disease.
2- Coxsackievirus A16 or enterovirus 71, although other Coxsackievirus
types and echoviruses have been implicated.
3- Aphthous ulcers. herpangina, herpetic gingivostomatitis, and thrush.
Slide 22
Slide 23
Slide 24

Erythema Multiforme Minor


Individual lesions last about 1 week.

The eruption may continue to appear in crops for 2 to 3 weeks.

Duration from onset to healing: 1 to 4 weeks

Lesions heal without scarring.


Slide 25

Streptococcal scarlet fever.


A-diffuse, blanching, erythematous rash that has a sandpapery consistency on
palpation.
B-a white strawberry tongue
C-Pastia lines
D-Desquamation fine, thin flakes as the acute phase of the illness resolves
Treatment with a 10-day course of penicillin or erythromycin (in penicillin-
allergic children) is important for reducing the risk of transmission and
preventing rheumatic fever and pyogenic mcomplications

Staphylococcal scarlet fever.


Slide 26
a child who is in shock (decreased blood pressure, increased heart rate,
lethargic with the following lesion
1. Diagnosis?
2. What is your approach?
3. What is the organism?
4. What antibiotics?

Answer

1. Meningococcemia
2. Cover shock management
3. N. meningitides
4. Pencilline
Slide 27
You are treating a 2-year-old girl who has suspected meningococcal
bacteremia and meningitis. Over the past 2 hours, she has required multiple
fluid boluses and inotropic support to help maintain her blood pressure. She
has been intubated due to respiratory failure. Her temperature is 96°F
(35.6°C), and she is covered in a petechial and purpuric rash. Her most recent
laboratory results reveal a white blood cell count of 1.2x103/mcL (1.2x109/L)
with 80% lymphocytes, 10% neutrophils, and 10% band forms and a platelet
count of 32x103/mcL (32x109/L).

1-What is the most likely cause of her laboratory results?


2- What is the most important additional laboratory test?

Answer
1- Disseminated intravascular coagulation (DIC) associated with septic shock.
2- Measurement of fibrinogen.
Slide 28
A 14-month-old girl is brought to the emergency department with a 12-hour
history of fever and rash. Her mother became frightened when it was difficult
to arouse the girl after her nap.
Findings on physical examination include a temperature of 104°F (40°C), a
heart rate of 164 beats/min, a respiratory rate of 42 breaths/min, and a blood
pressure of 75/45 mm Hg. There are petechiae and purpura on the chest,
arms, and legs.

1-What is the most likely diagnosis?


2-What is the most appropriate initial therapy?

Answer
1- Sepsis due to Neisseria meningitides.
2- Vancomycin plus ceftriaxone.
Slide 29
Slide 30
A 13-year-old boy presents to the emergency department with a 3-day history
of severe sore throat and fever. He is having trouble swallowing due to pain.

1-What Dx and the most likely cause of her laboratory results?


2- What is the most important additional laboratory test?

Answer
1- The bacteria that are involved most commonly are group A Streptococcus
and mixed oropharyngeal anaerobes.
2- Antimicrobial therapy must cover group A Streptococcus and oral
anaerobes, and clindamycin is a good choice.
Slide 31

Ecthyma gangrenosum

 caused by Septicemia with Pseudomonas aeruginosa


 Blood test show Neutropenia.
Slide 32

Staphylococcal scalded skin syndrome (SSSS)..


Complications include fluid and electrolyte losses leading to hypovolemia, faulty
temperature regulation, cutaneous infection (cellulitis), pneumonia, and
septicemia.

Overall mortality varies between 1 and 10%. Mortality in adults is about 60%
despite aggressive treatment, usually because of serious underlying illness.
Slide 33
Slide 34

Tetanus
1- caused by a neurotoxin from the anaerobic bacterium Clostridium tetani.

2- Treatment includes the use of human tetanus immune globulin (TIG), oral
antimicrobial agents (metronidazole or penicillin) for 10 to 14 days, and other
supportive measures (e.g., ventilator support, decreased external stimuli such as
loud noises). In addition, all infected wounds should be cleaned properly and
debrided.

3- Tetanus is prevented best through immunization with tetanus toxoid. Tetanus


toxoid is available:

1) in combination with diphtheria toxoid and acellular pertussis vaccine (DTaP) to


provide basic immunity against tetanus, diphtheria, and pertussis;

2) as part of a double antigen (DT) for children up to 6 years of age who cannot
receive the pertussis component of the DTaP; and

3) as a single antigen (tetanus toxoid) (TT) to immunize pregnant women and


women of childbearing age to prevent tetanus in their newborns.
Slide 35
An unimmunized 4-year-old girl presents with malaise, sore throat, and difficulty
swallowing. On physical examination, she has a temperature of 38.0°C, bilateral
cervical adenopathy, and grayish exudates over the mucous membranes of her
tonsils and pharynx.

When you attempt to remove some of the exudate for culture, bleeding occurs.

1-What is the most likely diagnosis?

2-How it is treated?

3-What is the most appropriate treatment of close contacts of this child?

Answer
1- Membranous pharyngitis caused by Corynebacterium diphtheria.

2- The mainstay of therapy is diphtheria antitoxin and antimicrobial therapy to stop


toxin production, eradicate the organism, and prevent transmission. Acceptable
regimens include erythromycin orally or IV for 14 days, penicillin G procaine
intramuscularly (IM) for 14 days, or penicillin G IM or IV for 14 days.

3- Close contacts of patients who have diphtheria, regardless of their


immunization status, should receive a single IM dose of penicillin G benzathine or
10 days of oral erythromycin.
Slide 36
A 3-year-old boy presents with a complaint of a swollen finger . He was
playing with the family cat yesterday, and the cat bit him. Within 24 hours, the
mother noted redness and swelling of the finger. Physical examination reveals
a temperature of 101°F (38.3°C) and an erythematous area surrounding two
puncture marks on the palm of his right hand. The palm is very tender to
touch. The mother reminds you that he is allergic to penicillin.

1-What are the most likely causative organisms?


2- What is the best choice of initial antimicrobial therapy for this child?
3- Where the infection rate is higher, in cat or dog bite and why?
Answer
1- Patients who have been bitten by a cat or dog and develop signs and
symptoms of infection within 24 to 48 hours most likely have an infection with
Pasteurella multocida.
2- Since the patient described is allergic to penicillin, which makes clindamycin
and trimethoprim-sulfamethoxazole the appropriate therapy. The clindamycin
is required for coverage of Staphylococcus aureus, which is common in animal
bites; the trimethoprim-sulfamethoxazole kills the Pasteurella.
3- The infection rate associated with cat bites is nearly double that of dog
bites because most cat bites result in puncture wounds, while dogs tend to rip
and tear the flesh when biting. Puncture wounds are more difficult to clean
and, therefore, are more likely to become infected.
Slide 37
A 4-month-old is brought to clinic by his parents for evaluation of bilateral droopy
eyes. His mother believes this has developed just over the last week. The child
recently started taking cereal in addition to breastfeeding and has been
constipated. Physical examination reveals droopy eyelids and 1+ deep tendon
reflexes diffusely.

1-What is the most likely diagnosis?

2-Enumerate 3 neuromuscular causes for ptosis?

Answer
1- The bilateral ptosis, combined with constipation and diminished reflexes, all
developing during the introduction of solid foods described for the infant in the
vignette is classic for botulism.

2- Neuromuscular causes for ptosis that have a new-onset neuropathic or


myopathic basis include myasthenia gravis, mitochondrial disease (chronic
progressive external ophthalmoplegia), and toxin exposures, such as botulism,
diphtheria, tick paralysis, insecticides, and vincristine.
Slide 38

Erysipelas
distinct infection of the skin involving the

uppermost layers of the subcutaneous tissue and cutaneous lymphatic vessels.

Erysipelas is caused by group A beta-hemolytic streptococci (GABHS) in the great


majority of the cases. Rarely it is caused by groups G, C, and B streptococci.

DDx

ct dermatitis

-like lesions of familial Mediterranean feverNatural history


Slide 39
A 13-year-old girl presents with a 2-day history of fever, sore throat, and a
rash that began on her arms and legs and spread to her chest and back.
Physical examination reveals pharyngeal exudate; bilateral cervical
adenopathy; and a "sandpapery" rash over her arms, legs, and trunk. A rapid
diagnostic test for group A Streptococcus yields negative results. At 48 hours,
a throat culture is growing small colonies with narrow bands of hemolysis on
sheep blood agar.

1-What is the most likely organism?


2- What is the most appropriate antibiotic for treating this patient?

Answer

1- Arcanobacterium (formerly Corynebacterium) haemolyticum.


2- Erythromycin is the drug of choice for treating pharyngitis
caused by A haemolyticum.
Slide 40
Slide 41

Tinea versicolor

superficial skin infection caused by the yeast Malassezia furfur (previously


known as Pityrosporum ovale or Pityrosporum orbiculare).

Predisposing factors (that allow lowering the body’s resistance and


proliferation of the yeast) include malnutrition, burns, immunosuppressive or
corticosteroid therapy, Cushing’s syndrome, and pregnancy.

Excess heat and humidity and oily skin or application of oils (e.g., cocoa
butter, bath oil) on children leads to yeast overgrowth.
Slide 42

Candida albicans,

which is a normal commensal organism in human.


The repeated courses of antibiotic have created the conditions for Candida
organisms to become pathogens.
The skin, throughout the entire gastrointestinal (GI) tract, in the female
genital tract, and in the urine of patients who have indwelling Foley
catheters.
Slide 43

1. Widespread fungal dermatitis with Candida albicans

Over the trunk (A) and foot (B) and nails (C).

With Severe combined immunodeficiency.


Slide 44
Slide 45
Slide 46

1. What is the diagnosis?


2. What are the three species associated with it?
3. What are useful methods for diagnosing tinea infections?

Answer

1. Tinea infection
2. Trichophyton
Microsporum
Epidermophyton
3. Potassium hydroxide (KOH) preparations
Fungal culture of skin scrapings inoculated on DTM(dermatophyte test media)
Slide 47
The pictures are of a 2 yrs old child who presented with intense pruritis ,
particularly at night. There are similar complaints seen in other family members.

1) What is the diagnosis?

2) What the characteristic finding for same?

3) What are complications seen?

4) What is the causative agent?

5) What is the complete treatment for this case

Answer

1) scabies

2) burrows

3) glomerulonephritis and pyoderma

4) sarcoptes scabies

5) permethrin 5% , topical corticosteroids for pruritis, treat entire family


Slide 48
3 yrs old girl who presented with intense pruritus , particularly at night and after
taking bath with worm water , pruritus is most significant in web space of hands
and toes

1) What is the diagnosis?


2) What the causative organism?
3) What are topical Rx?
4) What is oral Rx?

Answer
1. Scabies
2. contagious skin infestation caused by the female mite Sarcoptes scabiei.
3. Permethrin 5% cream (preferred treatment)
 Safe and effective
 Apply from the neck down (older children and adolescents) including
intertriginous and genital areas, the intergluteal cleft, and under trimmed
nails , head, scalp, and neck in infants and young children, treatment of
entire head, neck, and body in this age group is required (avoid areas
around the eyes and mouth).
 Leave the medication on overnight (about 8 hours).
 Change all the clothes and bed sheets the following morning.
 Repeat 1 week later for best results.
4. Ivermectin
Slide 49

1) What is the diagnosis?

2) Commonest organism?

3) Treatment

Answer for Question No : 76

1) Cutaneous larva migrans

2) Anky. Braziliense (Hook worm of dogs and cats)(other anky & Strongyloides)

3) Ivermectin - 200mg/kg/one 1 to 2 days

Albendazole - 1 X OD X 3 days

Topical thiabendazole
HEMATOLOGICAL SLIDES
1. blood film of acanthocytosis , betalipoprotienemia ,what system affected
2. blood film (ID Aneamia)
3. blood film of SCD
4. spherocytosis
5. Blood film of Hypercromic RBC with hypersegmental neutrophile.
6. blood film of target cell (4 examples)
7. Acute hemolytic anemia
8. HUS
9. blood film of Lead poisoning ,blood film (basophile stippling )
10. blood film of Toxic granulation
11. 9. blood film of ALL
12. blood film of AML
13. bone marrow aspirate of gaucher cell
14. bone marrow aspirate of leshmania donvani
Diagrammatic representation of different types of poikilocyte.
Normal periphral smear
Specific Red Cell Morphologic Abnormalities
Target cells Increased surface area /volume ratio
Cell with a strongly staining area in the centre of the area of central pallor

 Hb SS, SC, S-Thalassemia


 Thalassemia
 Hb AC or CC
 Hb E (heterozygote and homozygote)
 Severe iron deficiency
 Postsplenectomy or hyposplenic states
 Liver disease
 Abetalipoproteinemia
Spherocytes Decreased surface/volume ratio (>MCHC >35 g/dL)
Larg reticulocytes with small spherocytes result in normal MCV.

– Spherocyte: Cell which is approximately spherical in shape so that it has


lost its central pallor; the cell outline is regular
– Microspherocyte: densely stained RBCs with no central pallor
Seen in
 Hereditary spherocytosis
 Autoimmune hemolytic anemia
 Microangiopathic hemolytic anemia
 ABO incompatibility
 G6PD and Pyruvate kinase deficiency
 Hypersplenism
 Post transfusion
 thermal injury
 clostridial septicemia
 Wilson disease
Hereditary spherocytosis

Dx Incubated osmotic fragility

Treatment Folic acid , Splenectomy

The MCHC is only raised in


 hereditary spherocytosis
 hereditary xerocytosis
 hereditary pyropoikilocytosis
 SS anemia
 Cold agglutinin disease.
Elliptocytes
Elliptical cells, normochromic; seen normally in less than 1% of RBCs; red cells
that are dark and lack central pallor.
seen in various anemias, especially macrocytic ones. elliptocytosis, discloses
pencil cells” increased with severity, hypochromic - IDA

 a normal patient
 Hereditary elliptocytosis AD
 Iron deficiency (increased with severity, hypochromic)
 SC disease
 Thalassemia major
 Megaloblastic anemias
 Malaria
 Severe bacterial infection
Stomatocytes

Has a slit-like area of central pallor


 Normal (in small numbers)
 Hereditary stomatocytosis
 Thalassemia
 Acute alcoholism
 Liver disease
 Malignancies


Sickle cell:
Cell with a sickle or crescent shape, caused by the presence of a hig
concentration of an abnormal haemoglobin known as haemoglobin S result of
change encodes valine instead of glutamine in the 6th position in the β-globin
molecule
Sickle cell anemia - Autosomal recessive
Dx : Hemoglobin electrophoresis or HPLC
Complicated with Stroke
Treatment
Prophylaxis Penicillin
 Maintain adequate hydration
 Avoid hypoxia
 Folic acid supplements
 Blood transfusions
 BMT
 Hydroxy Urea
A 5 yr boy c/o pallor and passing dark-colored urine. Three days prior the
patient had low-grade fever and dry cough. Initial CBC revealed a Hemoglobin
level of 4.5 g/dL, an Hct of 14%, a WBC count of 9,100/mm3. (N76%, L14%,
M8%, E1%) and a platelet count of 372,000/mm3. The patient had been
diagnosed with Hemoglobin H disease since 18 months of age. He had
received 2 blood transfusions when he had fever and acute anemia. Both
parents are thalassemia carriers.
Physical examination: Temp- 38.3 C, Markedly pale
Heart: Syst. ejection murmur grade II/VI at lt upper sternum
Abdomen: Splenohepatomegaly

Hb H/Hb CS disease with hemolytic crisis


Acanthocytes (spur cells)
Cell with its surface covered with two to twenty projections (spicules) of
irregular shape and irregularly distributed
Most ofthe affected erythrocytes are also small and lack central pallor
 Abetalipoproteinemia
 Malabsorptive states
 Vitamin E deficiency
 Neuroacantocytosis
 Severe liver dysfunction
 Hypothyroidism
 Postsplenectomy or hyposplenic state

Abetalipoproteinemia - autosomal recessive


disorder of lipid metabolism ,decrease cholesterol and TGL and Absent B
Liproteins characterized by
 fat malabsorption
 FTT
 chronic Diarrhea
 ataxia
 Retinitis pigmentosa
Schistocytes Helmet, triangular shapes, or small fragments.
Caused by fragmentation on impact with abnormal vascular surface (e.g.,
fibrin strand, vasculitis, artificial surface in circulation)

 Disseminated intravascular coagulation (DIC)


 Hemolytic uremic syndrome
 Severe hemolytic anemia (e.g, G6PD deficiency)
 Prosthetic cardiac valve
 Thrombotic thrombocytopenia purpura (TTP)
 Kasabach–Merritt syndrome
 Purpura fulminans
 Renal vein thrombosis
 Burns (spheroschistocytes as a result of heat)
Acute hemolytic anemia
RBC fragments, anisocytosis, polychromasia, decreased platelets

Seen in
 hemolytic-uremic syndrome
 thrombotic thrombocytopenic purpura.
 Kasabach-Merritt syndrome
teardrop cells (dacrocyte)
shape of drop,usually microcytic, hypochromic

 Thalassemia major
 Myeloproliferative syndromes
 Leukoerythroblastic reaction
Echinocytes (burr cells)
10–30 spicules equal in size and evenly distributed over RBC surface; caused by
alteration in extracellular or intracellular environment
 Uremia
 Liver disease
 storage artifact if blood is kept in a tube for several hours before preparation
 Pyruvate kinase deficiency
 Peptic ulcer disease or gastric carcinoma
 Immediately after red cell transfusion
 Rare congenital anemias due to decreased intracellular potassium
Bite Cell (Degmacyte)
Bite cells have a semicircular defect in their edge that resembles a bite mark.
These defects occur when certain drugs cause oxidative destruction of
hemoglobin often in patients with
 cute hemolytic anemia
 G6PD
 Disseminated intravascular coagulation (DIC)
 Hemolytic uremic syndrome

See also Heinz bodies, which are erythrocyte inclusions of denatured Hb



Microcytic, Hypochromic Red Cells with few target cells
the erythrocytes are small and have markedly increased central
pallor, exceeding 1⁄3 the diameter of the red cell. The erythrocytes,
therefore, are microcytic (7.0 m in diameter) and hypochromic.
These features, which usually coexist,
The major causes are
 iron deficiency (Hct In infants and young children <27.5, in adults ≤28.0)
 thalassemias
 sideroblastic anemias
 anemia of chronic disease
 Lead poisoning

need
• Serum iron/TIBC
• Hemoglobin electrophoresis
• Bone marrow study
IDA

Hypochromic microcytic, anisocytosis, poiklocytosis,and target cells.


RDW >13, high serum ferritin, low serum iron, & high iron binding capacity
IDA
15 months old child admitted for evaluation with anemia ,no
hepatosplenomegaly, PS shows.

microcytic, hypochromic anemia, tear drops cells

DDx Lead poisoning/ sideroblastic anemia

Dx Serum ferritin

Rx Iron supplement
Thalassemia major
15 months old child admitted with anemia with hepatosplenomegaly for
evaluation, PS shows.

microcytic, hypochromic anemia with nucleated RBC, tear drops cells

Rx BMT
Basophilic stippling
Coarse or fine punctate basophilic inclusions that represent aggregates of
ribosomal RNA(Hypochromic microcytic anemia)

 Hemolytic anemias (e.g., thalassemia trait)


 Iron-deficiency anemia
 Lead poisoning
Howell–Jolly bodies
Small, well-defined, round, densely stained RBC -remnant RBC nuclear; 1 μm
indiameter; centric in location

 Postsplenectomy or hyposplenia
 Megaloblastic anemias
 Dyserythropoietic anemias
 rarely iron-deficiency anemia, hereditary spherocytosis
Heinz bodies Denatured aggregated hemoglobin
 G6PD defecincy
 Thalassemia
 Postsplenoectomy and Asplenia
 Chronic liver disease
Microcytic RBC (<75 FL)
 Iron deficiency
 α or β-Thalassemia trait
 Hemoglobin E disorders (AE, EE)
 Lead poisons
 Sidrobalstic
Macrocytic RBC (>85 FL)
– Vitamin B deficiency
12
– Folic acid deficiency
– Diamond-Blackfan anemia
– Pernicious anemia
– Malabsorption disorders
– Surgical resection of bowel
– drugs (e.g. phenytoin, phenobarbitone , methorexate,pryrimethamine)
– Alcohol excess
– Liver disease
– Hypothyroidism
– Myelodysplastic syndromes
Postsplenoectomy
• Macrocytes
• Thrombocytosis
• Target cells
• Acanthocytes
• Howell–Jolly bodies
• Heinz bodies

Hypersplenism
 Spherocytosis
Mature neutrophils or polymorphonuclear leukocyte
The nucleus normally contains condensed, dark-staining material arranged in
two to five lobes joined together ,half-life of neutrophil in circulation 6 hours
Band (Juvenile immature neutrophils)
They differ from mature neutrophils primarily in having a curved or coiled
nucleus, does not segment into lobes.
An increase in the number of bands and other immature neutrophils is called
a “shift to the left” and can occur in many situations, including
 infections
 noninfectious inflammatory diseases
 uncomplicated pregnancy.
Toxic Granulation
Indicates the presence of Prominent dark blue and increased numbers of
granules that are larger and more basophilic than normal. It may occur with
 severe infections (especially bacterial)
 Kawasaki disease.
 colony-stimulating factors
 burns
 malignancies
 drug reactions
 aplastic anemia
 the hypereosinophilic syndrome.
Hypersegmented neutrophil
The normal number of lobes in neutrophils is average about three.
Hypersegmentation exists when 5% of neutrophils have six or more
lobes. Such hypersegmentation is common in
 folate and vitamin B12 deficiency
 myelodysplastic and myeloproliferative disorders.

Also can see


 Neutropenia, and thrombocytopenia
 increased levels of lactate dehydrogenase, bilirubin, and iron in serum
 increased transferrin saturation, which reflects “ineffective
erythropoiesis.”
 Serum cholesterol, lipid, and immunoglobulin levels may be decreased.
Atypical Lymphocyte
are usually larger than small lymphocytes and have an oval, kidney-shaped, or
lobulated nucleus, which may appear folded. Nucleoli are sometimes
prominent, and the chromatin is coarse, reticular, or clumped

Common with
 Epstein-Barr virus (infectious mononucleosis)
 cytomegalovirus infection
 human immunodeficiency virus (HIV) infection.
 Toxoplasmosis
 drug reactions.
ALL
This is the bone marrow aspirate of a 5 yrs old child with a history of fever ,
bone pain , hepatosplenomegaly . BFS show Blasts—lymphoblasts

Common with
 Down syndrome
 Bloom syndrome
 ataxia-telangiectasia
 Fanconi’s syndrom
 Epstein-Barr viral infections
AML
This is smear of a 15 yrs old child with a history of fever , bone pain with
bleeding and hepatosplenomegaly.

Large immature cells , Two or more nucleoli , Auer rods- distinctive rod like
red staining structures

It has an increased incidence in

 Down's syndrome
 Fanconi anaemia
 Diamond-Blackfan anaemia
 Kostmann syndrome and
 Bloom syndrome.
Gaucher disease
2 yrs old child with MR admitted for evaluation , On examination he had
anemia with organomegaly ,erlenmeyer deformity, osteosclerosis, lytic lesion.
Lab evaluation showed anemia with thrombocytopenia .BM done .

macrophages are shown which have crumpled appearing cytoplasm and


eccentric nuclei, consistent with Gaucher cells.
The other hematopoietic elements present show normal maturation.

Wrinkled paper cells in BM , Intracytoplsmic substrate inclusion

Test : B glucosidase activity in peripheral leucocyte fibroblasts


DDx Niemann Pick disease, MPS, granulocytic leukemia and myeloma.,
Rx Enzyme replacement(acid beta glucosidase) 60 units/kg, IV on alternate
week and then monthly maintenance
bone marrow transplant.
Leishmaniasis
This is the bone marrow aspirate of a 15 month old child with a history of
prolonged fever , hepatosplenomegaly and pancytopenia

Bone marrow smear showing Amastigote form of Leishmania donovani , Kala


Azar present inside macrophages in the bone marrow.
R x Sodium stilbogluconate IM/IV 20 mg/kg/day for 30 days.
pentamidine ,Amphotericin B, oral miltefosine
Falciparum malaria
2 yrs old child admitted with anemia , hepatosplenomegaly and fever. PBS
shows :

Rx quinine iv
Acid-fast bacilli

Ziehl neelsen technique CSEWM


Pneumocystis carinii

Rx

Cotrimoxazole 5mg/kg once daily 3 day a week

15 – 20mg in 4 weeks for AIDS

for others Pentamidine ,Atovaquone, trimextrate + steroids 2 weeks


Diphtheria
Giardia Lamblia –Trophozoite

RX Antiprotozoal agent
Albendazole (albenza) 400mg once a day for 5 days PO
Furazolidone (Furoxone) 6 mg/kg/24 hr divided qid PO for 10 days
(maximum : 400mg/24 hr)
Metronidazole (Flagyl) 15mg/kg/24hr divided tid PO for 5 days
(maximum : 750mg/24 hr)
Paromomycin Not recommended
Quinacrine (Atabrine) 6mg/kg/24 hr divided tid PO for 5 days
(maximum : 300mg/24 hr)
Tinidazole 50mg/kg once(maximum :2g) Not available in USA .
Egg of Ascaris Lumbricoides

Rx : Albendazole 400 mg or Mebendazole 100 mg BD x 3 days 6B:


Ascaris Ova

Rx
1. a) Piperazine citrate 150 mg / kg stat Follwed by 6 doses of 65mg / kg Bd
2. Loeffler’s Syndrome 10 to 24 Months
E. Histolytica

Rx

1. nitroimidazole, metronidazole, chloroquine


Hookworm egg

C/P ; Iron deficiency anemia, dermatitis, cutaneous larva migrans,


cough due to laryngotracheobronchitis.
Rx : Albendazole 400mg stat, Mebendazole 100mg BD for 3 days,
pyrantel palmoate 11mg/kg OD for 3 days.
RADIOLOGICAL
SLIDES
Chest x ray
1. Chest X ray (TGA)
2. Chest X ray (TAPVD)
3. Chest X ray (TOF)
4. Chest X ray (CMP)
5. Chest x ray pnemothorax …Rx
6. Chest x ray diaphragmatic hernia /R x what u should avoid in mx of this baby
7. Chest x ray Foreign body
8. Chest x ray pleural effusion
9. Chest X ray (RDS-HMD)
10. Chest x ray lobar pneumonia (……)
11. Chest x ray pulmonary abscess(empyma)
12. Chest X ray - congenital emphysema
13. Chest x ray - Pneumatocele
14. Chest x ray finding and Dx ,abscent clavicle (cleidocarinal dysostosis,AD)
15. Chest X ray (croup)
16. Lateral neck x ray "thumb" sign / Epiglotitis
Abdomen x ray and graphy
1. abdomen x ray with distended loops; differential , how to prevent it (NEC)
2. abdomen x ray deudeal atresia
3. abdomen x ray Nephrocalcinosis
4. B.enema – coil ,spring signs (intussuscption disease)
5. B.enema hirshprung disease
6. IVU
Hand and legs x ray
1. hand x ray cystenosis
2. hand x ray /CRF
3. leg x ray Vitamin D deficiency
4. leg x ray perths disease
CT and MRI
1. brain CT / SAH
2. brain CT/ Epidural hematoma
3. Brain Ct /subdural HRG
4. Brain Ct / cerebral infarction
5. Brain Ct / CMV
6. brain CT/ Dandy-walker syndrome
7. brain MRI (abscess stages)
8. Abdominal CT / -Describe what you see ,-What is the diagnosis? Wilms tumor
Radiology -X ray
RADIOLOGY 1

1. Diagnosis?
2. What are the first two steps in treatment of hypoxic spell?
3. In a cyanotic newborn, how can you distinguish pulmonary disease from
cyanotic congenital heart disease?
4. most common neurological complications associate with disease
5. Which cardiac conditions are associated with following
Egg Shaped Heart
Snowman silhouette
Rib notching

Answers
1.cyanotic congenital heart diseseas ,most probably TOF
Pulmonary vascular markings are decreased, low PBF
RVH on with upturn of apex
A hypoplastic main pulmonary artery segment contributes to the formation
of the “Coeur en sabot” “boot-shaped” heart.

2.knee chest position , morphine

3.Hyperoxia Test

4.Cerebral thrombosis, and brain abscess.


5.X-ray appearances
 Egg Shaped Heart Transposition of great arteries
 Snowman silhouette TAPVD (supracardiac)
 Rib notching Co-arctation of aorta (long standing)
RADIOLOGY 2

1. What is this sign called?


2. What are the structures that cause this appearance in this condition?
3. What is the diagnosis?
4. What are the treatment?

Answer

1. i.The heart shape is ovoid (egg-shaped).


ii. The upper mediastinum is narrow.
iii. The lung fields appear normal , no VSD.
2. Transposition of the great arteries.
The diagnosis should be confirmed by echocardiography.
A prostaglandin infusion should be started and the case should be discussed
with the nearest pediatric cardiac centre as soon as possible.
RADIOLOGY3

Chest X ray of baby IDMM

1. Describe the X-ray.


2. What is your diagnosis?
3. What will you do next?

Answer

1.The heart is much larger than normal.


The lung fields appear well aerated.
UAC on the left – very high
UVC on the right, very high

2. A diabetic cardiomyopathy is the most likely diagnosis.


Also baby IDMM may have TGA

3.Rx Reposition the lines.

Echocardiography.

Seek expert advice if haemodynamically unstable.


RADIOLOGY4

1. What is the diagnosis?


2. What is this sign called?
3. What are the structures that cause this appearance in this condition?
4. What are the clinical presentation of the infant?

Answer

1. Total anomalous pulmonary venous drainage- supracardiac type

2. Snow man appearance, figure of 8 appearance

3.Cardiomegaly with increased vascular markings

Dilatation of both the left and right innominate veins and right SVC .

enlargement of sup. mediastinum secondary to right SVC, innominate artery

and ascending vertical vein.

4. Mild cyanosis, cardiac failure, recurrent chest infection, pulmonary HT.


RADIOLOGY5

1. What was the lesion?


2. What was the procedure carried out?
3. What are the complications ?

Answer

1. VSD
2. Device implant
3. Device displacement
Emboli formation
Haemolytic anaemia
RADIOLOGY6

1. Identify the procedure, where the device shown in this X-ray is used.
2. List two indications of this procedure.
3. What is the long-term non-cardiac complication, which may occur, if
the device is not placed in asymptomatic children?

Answer
1. ASD device closure
2. All symptomatic patients:
Asymptomatic patients with a Qp : Qs ratio of at least 2 : 1
3. Paradoxical (right to left shunt )
systemic embolization.
RADIOLOGY7

1. Identify the procedure, where the device shown in this X-ray is used.
2. List five complications, which may occur, if the device is not placed in
children with the above diagnosis.

Answer
1. PDA coil closure
2. Cardiac failure:
Infective endarteritis
Aneurysmal dilatation of the pulmonary artery
Calcification of the ductus
Non-infective thrombosis of the ductus with embolization
Paradoxical emboli
Pulmonary hypertension.
RADIOLOGY8

1) What is abnormalities , your diagnosis ?


2) Name three risk factor for this diagnosis?
3) What is the treatment?
4) What is the dose?

Answer
1. Respiratory distress syndrome
widespread opacification throughout both lung fields.
There are clear air bronchograms on both sides.
The heart border is not clearly defined.
The costophrenic and cardiophrenic angles are not clearly visualised.
There is an endotracheal tube.
2. Preterm, male, elective LSCS, gestational diabetes, multiple gestation, asphyxia
3. R x Antenatal corticosteroids - Betamethasone
Betamethasone 12 mg /12hour interval/I.M. /24 hours prior to delivery
Surfactant should be given if the dose has not been repeated since birth.
Ventilatory requirements are high and some centers
would consider high frequency oscillatory ventilation at this point. If this is not
available ventilation will probably need to be adjusted to improve the blood gases.
RADIOLOGY9

1. Describe the X –ray of post term NB?


2. Diagnosis?
3. Give three recent advances in management ?

Answer

1. Hyperinflated lung with diffused patch infiltration , and atelectasis


2. MAS
3. Lung lavage , surfactant instillation , HFOV
RADIOLOGY10

1) Identify the condition in the CXR of an ELBW newborn


2) Give the definition of this condition
3) Mention the stages of this condition in a 34 wk old
4) What are the pharmacological strategies in the management of this condition
5) Expand INSURE

Answer
1. Bronchopulmonary dysplasia (BPD)
2. Current definitions include
total duration of oxygen supplementation requirement for >28 days,
degree of prematurity (<32 weeks gestational age at birth), and
Oxygen dependency at 36 weeks Postmenstrual age.
3. Stages:
a. Mild: Breathing room air at 56 days postnatal age or discharge*
b. Moderate: Need for <30% oxygen at 56 days postnatal age or discharge*
c. Severe: Need for > 30% oxygen and/or positive pressure (IMV/CPAP) at
56 days postnatal age or discharge* (* whichever comes first)
4. Pharmacological strategies
a. Vitamin A
b. Postnatal steroids
c. Superoxide dismutase
d. Furosemide
e. Intubate SURfactant Extubate
RADIOLOGY11
RADIOLOGY12

1. Diagnosis?

2. What is the clinical picture?

3. What is the requirement of echo before surgery?

Answers

1. Tracheo-esophageal fistula

2. Excessive drooling ,Respiratory distress

3. To rule out associated Congenital heart diseases and Right sided aorta
RADIOLOGY13
1. What is the abnormality?
2. What is the diagnosis ?
3. How is it suspected clinically ?
4. What is the management ?
5. What are three causes of respiratory distress in a baby born with this condition?

Answer
1. Bowel loops in Right hemithorax, mediastinal shift to left
2. Congenital Right diaphragmatic hernia
3. Respiratory distress
Mediastinal shift
Bowel sounds in the thorax
Scaphoid abdomen
4. no baging using a mask and mask ventilation avoiding lung inflation prevent the
bowel from distending any more with swallowed air.
NGT on drainage
Intubation and ventilation immediately after birth till pt stabilizes
Refer to NICU
Treat PPHN
5. Surgical correction A pediatric surgical opinion should be sought.
1. a) Mechanical compression of the lungs from the herniated viscera
b) Pulmonary hypoplasia from compression of the developing lungs in utero
c) Pulmonary hypertension
RADIOLOGY14

1. Diagnosis ?

2. Differential diagnosis of this Xray ?

3. Maternal condition associated ?

4. Prognostic factors associated with better outcome ?

Answer

1. Diagnosis: Congenital left Diaphragmatic Hernia (CDH)

2. Differential diagnosis of this Xray:

-Congenital cystic adenomatoid malformation (CCAM)

-Cystic pulmonary interstitial emphysema

-Staphylococcal pneumonia with pneumatocele formation.

3. Maternal condition associated: Polyhydramnios

4. Prognostic factors associated with better outcome:

-Herniation after 2nd trimester

-Absence of liver herniation

-Late onset of postnatal symptoms


RADIOLOGY15

Congenital left diaphragmatic hernia


RADIOLOGY16

Left Congenital lobar emphysema

Hyperinflation of the left upper lobe, paucity of vascular markings of the left
upper lobe, mediastinal shift to the right, atelectasis of the left lower lobe,
flattening of the left hemidiaphragm.

Common site of involvement In Left upper lobe.


RADIOLOGY17

Congenital left upper lobe emphysema.

Extension of the emphysematous lobe into the left lower lobe and its
displacement of the mediastinum toward the right.

In 50% of cases, a cause of CLE can be identified.


 Congenital deficiency of the bronchial cartilage
 external compression by aberrant vessels
 bronchial stenosis
 redundant bronchial mucosal flaps
 kinking of the bronchus.
RADIOLOGY18
RADIOLOGY19

Describe chest xray ,whts Rx ?

Huge a left tension pneumothorax with mediastinal shift to the right.

endotracheal tube that is slightly high.

There is a central venous line with the tip at the thoracic inlet.

There is an umbilical venous line with a very low tip.

Rx A chest drain must be inserted immediately


RADIOLOGY20

Describe chest xray ,whts Rx ?

Large tension pneumothorax on the left.

Mediastinal shift to the right with tracheal deviation.

Transcutaneous oxygen electrode on left upper chest.

Rx Immediate drainage of the pneumothorax, needle in 2nd LT ICS then drain

Intubation and ventilation is very likely to be needed.


RADIOLOGY21
RADIOLOGY22

1. What is the radiological diagnosis?

2. Name 2 risk factors for the development of this condition

Answer

1. Right pneumothorax mediastinal shift to left with left upper zone haziness

2. Risk Factors-

• Assisted ventilation (including CPAP)

• MAS

• RDS

• Other Air Leak Syndromes (e.g. PIE)

• Pulmonary hypoplasia

• CHD

• Idiopathic or spontaneous
RADIOLOGY23

1. What is the abnormality


2. Three high risk situations when this condition is imminent
3. Management

Answer

1. Pneumothorax
2. Put a needle in second intercostal space then Intercostal drain(done)
RADIOLOGY24

1. Describe the X ray findings .


2. What is the diagnosis?
3. What is the treatment of choice ?
4. Write 4 life threatening complications of Kawasaki disease.
5. Write side effects pertaining to CVS of Digitalis

Answer

1. X ray chest PA view with air trapped s/o


2. Pneumopericrdium
3. O2 and monitoring if severe distress prompt evacuation.
4. MI, Coranry aneurusm, Thrombosis, DIC
5. Atrial, ventricular extrasystole, heart block,AV block, VT,VF
RADIOLOGY25

1. Identify the abnormality?


2. Clinical presentation?
3. Treatment ?

Answer

1. Pneumopericardium
2. Shock with weak pulses
3. Drainage
RADIOLOGY26

1. What is the abnormality ?


2. What is it a complication of ?

Answer
1. Pneumomediastinum
2. Forceful ventilation
RADIOLOGY27

Autosomal dominant.

It results from a mutation in the CBFA1 gene, which controls a key


transcription factor in osteoblast differentiation.

The anterior fontanelle often closes late , may be delayed eruption of teeth.

There can be bossing of the forehead.


RADIOLOGY28

A 17-year-old boy states that he is healthy, although he admits to being


treated for three cases of left lower lobe pneumonia over the past 10 years.

1. Diagnosis?
2. What is the treatment ?

Answers

1. Pulmonary sequestration.
2. Surgical lobectomy generally is curative.
RADIOLOGY29

1. What is the X ray suggestive of ?


2. What is the likely organism?
3. What are the complications?
4. What is the drug of choice?

Answer

1. Lobar Pneumonia
2. Pnemococous/Staphylococcus
3. pneumonic effusion, empyema
4. Penicillin for susceptible org and cefotaxime / vancomycin for penicillin
resistant org for 10-14 days
RADIOLOGY30

Consolidation in the right lower lobe , S. pneumoniae.

indications for admission in this patient?

 Immunocompromised state
 Toxic appearance
 Moderate to severe respiratory distress
 Requirement for supplemental oxygen
 Complicated pneumonia
 Dehydration
 Vomiting or inability to tolerate oral fluids or medications
 No response to appropriate oral antibiotic therapy
 Social factors (e.g., inability of caregivers to administer medications at
home or follow-up appropriately)
RADIOLOGY31

If patient presents with fever and toxaemia.

1. give three differential diagnosis


2. Give three modalities of management

Answer

1. differential diagnosis
Lung Abcess
Infected Bronchogenic cyst
Infected Hydatid Cyst
2. management
Antibiotics (anaerobic +aerobic)
Chest physiotherapy
Percutaneous CT guided aspiration
RADIOLOGY32
6-year-old boy, who is presented with high fever, respiratory distress, and hypoxia. He
had an infected varicella lesion on his ear.

1. What are the CXR findings?

2. What is the most likely diagnosis?

3. What is your immediate management?

Answer

1. Complete opacification of the left lung field and a mediastinal shift to right.

2. Empyema

3. Chest tube insertion and antibiotics.


RADIOLOGY33

1. What is the diagnosis?


2. plural fluid finding in bacterial pneumonia ?
3. What does VATS stand for?

Answers
1. Pleural Effusion (Right)

2. plural fluid finding / exudative pleural fluid

 Proteins > 3.0 g/dL


 Pleural Fluid LDH > 200 IU/L
 Fluid to serum LDH ratio > 0.6
 Cell count > 1000
3. Video Assisted thoracoscopy
RADIOLOGY34

17 month old boy brought with H/o ingesting Kerosene.


First X ray( A) was taken at 3 hours and second (B) after few hours.
1. What is the role of gastric aspiration here on admission?
2. Ingestion of what amount is considered at risk for Pneumonitis ?
3. How long would observe this child ,if no abnormal symptoms develop.

Answer
1. Not to be done.
2. 30 ml
3. 8 – 12 hours
RADIOLOGY35

7-month-old child with worsening cough. On examination his temperature is


37.8oC. Respiratory rate 60 bpm, heart rate 130 bpm. He has moderate
intercostal recessions and his arterial oxygen saturations are 88% on room air.
Auscultation of his chest reveals bilateral scattered fine crepts. weight is 7.3
kg (10th percentile), and length and head circumference are at the 95th
percentile for age.

1. Finding ?
2. Diagnosis?
3. What is the treatment (mention complete schedule)?

Answers

1. Interstitial infiltrates beginning in the perihilar region and spreading to


the periphery. Apices spared until later in the disease.
2. Pneumocystis pneumonia.
RADIOLOGY36

4. Finding ?
5. Diagnosis?
6. What is the treatment (mention complete schedule)?

Answers

1. reticulonodular infiltrate distributed fairly uniformly throughout the lungs


,reflect nodular interstitial spread without alveolar involvement
2. Miliary tuberculosis
3. 2HRZE + 7HR
RADIOLOGY37

1. Xray finding?
2. DX ?
3. DDx?

Answer

1. thin-walled, air-filled cysts within the RT lung parenchyma


2. Pneumatoceles
3. Staphylococcus Aureus Infection
Bronchogenic Cyst
Cystic Adenomatoid Malformation
Hyperimmunoglobulinemia E (Job) Syndrome
Pneumococcal Infections
Pneumonia
Pulmonary Sequestration
Tuberculosis
Pneumatoceles are thin-walled, air-filled cysts that develop within the lung
parenchyma frequent with staphylococcal infections, and they should not be
confused with a pulmonary abscess.
Pneumatoceles have thin, smooth walls and are seen with an improving
clinical picture thought to be a form of localized pulmonary interstitial
emphysema and are self limiting with only the rare case of a large, persisting
pneumatocele needing surgery, whereas pulmonary abscesses have thick, irregular
walls with an air fluid level and the child tends to be very ill.
RADIOLOGY38
1. List 4 findings on the x- ray ?
2. What is the radiological diagnosis?
3. List 4 causes for the finding in the left lung ?

Answer
1. a) Mediastinal shift (L)
b) Pneumothorax (L)
c) Atlectasis lung (L)
d) Bronchopnemonic changes (R)

2. a) Pneumothorax
b) Bronchopneumonic changes (R) (Non – homogenous opacities)

3 . a) Pneumonia
b) Asthma
c) Foreign body in lung
d) Trauma
RADIOLOGY39

1. Mention the abnormality in this CXR.


2. Mention 3 causes of this appearance.
3. What further investigations would you order?

ANSWER

1. Mediastinal enlargement
2. Lymphoma
Thymoma
Teratoma
3. CT chest,CT guided biopsy and HPE
RADIOLOGY40

This the Chest X ray of an 11 year old female child with h/o recurrent lower
respiratory infections.

1)What is the diagnosis?

2)Write the a) clinical features and b) one important diagnostic clinical sign

3)Which syndrome is associated with the above condition?

4)Write the management

5)What is investigation of choice

Answer
1)Bronchiectasis
2)a) Productive cough with expectoration Hemoptysis ,FTT ,Cyanosis
b) Chest Deformities (Harrison's sulci) ,Crepitations, wheeze, crackles may
be heard on auscultation ,Clubbing
3) Kartagener's syndrome may be associated.
4) Management-
i. Treatment of underlying disorder
ii. Postural drainage
iii. Chest Physiotherapy
iv. Antibiotics
v. Surgical removal of the affected area
5) HRCT
RADIOLOGY41

1. What is the anatomical structure in which coin is lodged?

2. What is the location of carina with respect to thoracic vertebrae?

3. What are the anatomical areas of esophageal narrowing?

4. How can this foreign body be removed?

5. What complications you expect?

Answers

1. Esophagus (When foreign bodies lodge in the esophagus, the flat surface of
the object is seen in the AP view)
2. T 4
3. Anatomic areas of esophageal narrowing
a. Cricoid
b. Tracheal bifurcation
c. Gastro-esophageal junction
4. Emergent Endoscopy

5. Airway compromise.

Esophageal rupture.

Erosion into the mediastinal structures.


RADIOLOGY42

Child admitted with sudden breathing problems . There was history of playing
with marbles at the time of development of marbles. X-RAY done shows ?

1. Describe X RAY
2. Diagnosis
3. Treatment

Answer

1. X-Ray findings:-
The right lung volume is increased and has herniated across the mid-line.
The left lung is compressed by the displaced heart and mediastinum.
The left lung remains airated and normal bronchi are seen on that side.
The right main bronchus cannot be traced from its origin.
2. Rt main bronchus partially obstructed by non opaque foreign body
3. Bronchoscopy and removal
RADIOLOGY43

Three year old toxic boy with fever, drooling, stridor, respiratory distress.

1.what Xray finding?

2. what Diagnosis ?

3. what Treatment ?

Answer

1. Xray finding:

-Increased space between the pharyngeal air shadow and the vertebrae.

-Posterior pharyngeal wall is bulging

2. Diagnosis: Retropharyngeal Abscess

3. Treatment:

- Intravenous antibiotics with or without surgical drainage.

A third generation cephalosporin with ampicillin-sulbactam or clindamycin to

provide anaerobic coverage.


RADIOLOGY44

Scenario: this is a 5 years old child who presented to you with history of cough
and fever for 4 days who got lethargic, his vaccine history is incomplete, this
hisX-ray

Q1: Describe the finding?

Q2: What is your diagnosis?

Q3: How would you approach this child?

Q4: what is the possible organism?

Q5: What is other consideration you need to make?

Answer

1. Lateral view of the neck with "thumb" sign


2. Epiglottitis
3. ABC, don't do throat exam, Antibiotics, consult anesthesia for possible
emergency tracheostomy, oxygen, IV fluid
4. H. Influenza
5. To give prophylaxis for the family contact
RADIOLOGY45

1. What is the x-ray finding?

2. What is the most likely diagnosis?

3. What are the Indications for hospitalization?

Answer

1. AP radiograph of the neck showing typical steeple sign (subglottic narrowing).

2. Viral croup (Laryngotracheobronchitis).

3. Indications for hospitalization include:


 Persistent or worsening signs of respiratory distress despite therapy
 Signs of impending or frank respiratory failure or compensated respiratory failure
 Stridor at rest
 Unreliable caretaker
 Poor oral fluid intake
RADIOLOGY46

3 day neonate with Lethargy Feed refusal Abdominal distension (see x ray)
1. Describe the abnormalities on the X-ray ?
2. What is the diagnosis?
3. What is the radiological feature of Bell stage III NEC?
4. Name other conditions associated with pneumatosis intestinalis?

Answers
1. dilated bowel with thickened wall no air in the rectum
widespread intramural gas ,Pneumoperitoneum
2. NEC
3. Hirschsprung's disease
Pseudomembranous enterocolitis
Neonatal ulcerative colitis
Ischemic bowel disease
Bell’s staging criteria
Stage I (suspected NEC)
 Normal or mild dilatation or ileus
Stage II (definite NEC)
 Intestinal dilatation and pneumatosis intestinalis (subserosal,submucosal air)
Stage III (advanced NEC, severely ill)
 Same as II with portal vein gas and pneumoperitoneum
RADIOLOGY47

3 day neonate with Lethargy Feed refusal abdominal distension (see x ray)
1. What you see in this film?

2. What is your diagnosis?

3. What is the most likely cause in this preterm newborn?

Answers
1. Air under the diaphragm

2. Pneumoperitonium

3. Necrotizing enterocolitis
RADIOLOGY48
Answer Following questions based on X Ray seen
1) What is abnormal in this X ray?
2) What is the ideal position of placement of umbilical arterial and U venous line?
2. After putting in a UA line, the right lower limb appears pale ,what would you do?
3. What is the level of the renal artery?
4. How do you maintain a UA line?

Answers
1) Abnormally placed umbilical arterial line in the subclavian artery
2) For umbilical arterial line - High: Between T7- T10; Low: Between L2-L3
For umbilical vein - Just above the diaphragm
3) Warm the other limb; If still pale >1/2 hour, remove the UA line
4) L-1
5) Use heparin infusion at rate of 0.5-1.0 Unit per hour
RADIOLOGY49

Single bubble -Pyloric atresia


RADIOLOGY50

1. What are two imp radiological abnormalities?


2. What is the diagnosis ?
3. What are the three imp investigations ?
4. What are the metabolic abnormalities expected?
5. What is the management ?

Answer

1. Distension of stomach ,No gas in the intestines


2. Pyloric Stenosis
3. USS abdomen: pyloric thickness >4mm/ pyloric length >14 mm
S electrolytes
Ba studies

4. Hypochloremic, hypokalemic, metabolic alkalosis

5. Management of the fluid & electrolytes + Ramstedt's operation


RADIOLOGY51

Double bubble- Duodenal atresia


RADIOLOGY52

Triple bubble- Jejunal atresia


RADIOLOGY53

1. What is the abnormality?


2. What is the likely diagnosis?
3. Delineate management.
4. Mention 3 complications

Answer
1. Multiple fluid levels ,gasless lower abdomen
2. Small bowel obstruction
3. Surgical correction
4. Dyselectrolytemia
Perforation
Exaggerated hyperbilirubinemia
Dysmotility syndrome
RADIOLOGY54
1. What is the diagnosis?
2. Describe three features seen on the X-ray of the disease?
3. What biochemical test would help clinch the diagnosis?
4. What is the treatment of the condition?

Answers
1. Rickets
2. features seen on the X-ray
a) Cupping
b) Widening of distal end of metaphysis
c) Fraying
3. Calcium, Phosphorus, Alkaline phosphatase
4. Injection Vitamin D 6 lac unit IM stat and PO Calcium
RADIOLOGY55
1. What is the diagnosis?
2. This infant is 8 months old, what is the most likely type?
3. What is the earliest sign of this disorder?
4. What is the first radiological change that occurs in response to specific Rx?
5. How could this have been prevented?
6. What are the non - specific urinary findings in this disorder ? (at least 2)

Answer
1. Rickets
2. Vitamin D deficiency
3. Craniotabes
4. Appearance of provisional zone of calcification
5. Supplement of 400IU of vitamin D
6. Generalized aminoaciduria
> Glycosuria
> Phosphaturia
> Elevated urinary citrate
> Impaired renal acidification.
RADIOLOGY56

Short mother brings in her 1-year-old boy for the first time because she is
concerned about his “bowed legs”.
Radiographs of the boy’s long bones are obtained.

1-What is the most likely diagnosis?


2-What are the most likely serum laboratory findings?

Answer

1- Familial hypophosphatemic rickets of either the autosomal dominant or


sex-linked type.

2- The typical laboratory findings in this disorder are normal serum calcium
and low serum phosphate values.
RADIOLOGY57
RADIOLOGY58

1. what is the test you will ask for?


2. what is the treatment in this case?

Answer

1. Serum Ca+ and phosphate level


2. Vitamin D with Ca+ supplement
RADIOLOGY59

6 weeks infant K/C of Cholestatic jaundice (Extra-Hepatic Biliary Atresia)

• c/o swelling left thigh

1. What is the likely cause of fracture femur in this case?

2. How can this complication be prevented?

3. How do you manage pruritus in these patients?

4. An infant with cholestasis, triangular facies, and a pulmonic stenosis


murmur is likely to have what syndrome?

Answers

1. Metabolic Bone disease (secondary to Vitamin D deficiency due to


malabsorption of fat soluble vitamins)

2. Replace 5,000-8,000 U /d of D 2, or 3 -5 µg/kq/d of 25 hydroxycholecalciferol

3. Ursodeoxycholic acid 15-20 mg/kg/day

4. Alagille syndrome (Arteriohepatic dysplasia)


RADIOLOGY60

• Give five radiological findings ?

• What is the diagnosis ?

• What is the management ?

ANSWER

1. Ground glass appearance of bone


 Thinned cortex
 Periosteal calcification
 White line of Fraenkel (well calcified cartilage)
 Wimberger's sign (white ring)
2. Scurvy

3. Vit C 100-200 mg/ day + Dietary Therapy


RADIOLOGY61

X ray pictures of a 11 year old boy presenting with recurrent long bone
fractures
1. Identify the condition?
2. Mode of inheritance ?
3. Underlying pathology?
4. Mention 1 differential diagnosis
5. Other clinical Features in this condition? (Any 4)

Answers
1. Pyknodysostosis
2. Autosomal recessive
3. Lysosomal disorder due to genetic deficiency of Cathepsin K, which is
important for normal osteoclast function
4. Osteopetrosis
5. Short stature, Delayed closure of cranial sutures, fronto-parietal bossing,
short broad hands with hypoplasia of nails, nasal beaking, proptosis,
obtuse mandibular gonial angle
RADIOLOGY62

1. Findings in this x ray ?


2. What are the two important conditions which produce similar findings?
3. How do you differentiate radiologically these two conditions ?
4. What hematological problems can occur in a child with such x-ray findings ?

Answer
1. - Increased density of bone
- Changes suggestive of of rickets
2. - Osteopetrosis
- Pyknodysostosis
3. - Angle of mandible normal in osteopetrosis
- Increased angle of mandible in pyknodysostosis
- Distal phalanges normal in osteopetrosis
- Narrow distal phalanges in pyknodysostosis.
5. Anemia
RADIOLOGY63
1) Identify this condition.
2) Mention 3 treatment options
3) Mention 2 complications

ANSWER

1. Dense bones suggestive of osteopetrosis


2. Steroids,Interferon,Bone marrow Transplantation
3. Infections,Bleeds,Loss of vision
RADIOLOGY64
RADIOLOGY65
16-year-old boy who has a 3-month history of bilateral leg pain and lower back
pain. He also reports occasional low-grade fever and a 5- to 10-lb (2.25- to 4.5-kg)
weight loss over the last 3 months. He denies rashes or other symptoms.
His physical examination reveals loss of mobility of the lower spine when bending
forward and tenderness of both knees, hip radiograph was ordered.

1- What is the most likely diagnosis?


2- What other features you may see in this patient?
3- What finding you see in the hip radiograph?

answer
1- Ankylosing spondylitis, one of the diseases categorized as spondyloarthropathies,
or more recently, enthesitis-related arthritides.
2- Other features, including anterior uveitis, renal involvement, and rarely aortic
insufficiency, have been described in adolescents who have this disease.
3- Hip radiographs show evidence of sacroiliitis.
RADIOLOGY66

1. Describe the findings on the spine ?

2. Name the disease where this is seen?

3. Name 2 other skeletal complications of this condition ?

Answer
1. Infarction affecting the central part of the vertebrae (fed by a spinal artery branch)
results in the characteristic H. vertebrae of sickle cell disease. The outer portions of
the plates are spared because of the numerous apophyseal arteries.
The lateral cxray shows multiple vertebral end-plate irregularities and depressions.
The peripheral portions of the end-plates are spared.
The appearance is due to bone infarction and subsequent collapse. This is an early
example of the classical h-shaped vertebrae seen in sickle-cell anaemia.
2. Sickle cell anemia (also seen in Gauchers disease)
3. Dactylitis, avascular necrosis of femoral head and humerus, osteomyelitis
RADIOLOGY67

Baby 4 wks old girl never seen her turn her head, which makes it difficult for
her to feed at the breast. They also note that her back does not appear
normal. On physical examination, her hairline appears low posteriorly. You
confirm that she does not turn her head, and when placed prone, does not
turn her head to the side. Her right scapula appears to be higher than the left,
and you note that the spine does not appear to be perfectly straight,
suggesting congenital scoliosis x ray done .
1. Diagnosis?
2. What other associated defects may be seen??

Answers

1- The Klippel-Feil syndrome involves the fusion of cervical vertebrae and


occurs in approximately 1 in 42,000 births, with a 65% female predominance. It
is usually a sporadic event. Due to neck immobility, affected individuals are at
risk of cervical spine injury.
2-Associated defects may include deafness (conduction or
sensorineural, occurring in up to 30% of patients), congenital heart
defects (usually ventricular septal defect), rib defects,hemivertebrae, Sprengel
anomaly (elevation of the scapula), scoliosis, and renal anomalies.
RADIOLOGY68

1. Describe the lesion?


2. Give two D/D
3. What is the triad of tumor lysis syndrome?

Answers

1. Osteolytic lesion of skull


2. Histiocytosis
Ewing`s Sarcoma
Lymphoma
Bone cyst
3. Hyperuricemia, hyperkalemia, and hyperphosphatemia
RADIOLOGY69
1. What is the abnormality seen on this radiograph?
2. Name 2 disorders which may give rise to this abnormality

ANSWER

1. Xray skull lateral view showing multiple lytic lesions


2. Histiocytosis
3. Secondaries
4. Multiple myeloma
RADIOLOGY70

1. Describe the X-ray appearance


2. Pathogenesis of the appearance
3. Possible Diagnosis
4. Which disorder is most commonly associated with an elevated MCHC?
5. How is the corrected reticulocyte count calculated?

Answers

1. Sunray appearance
2. Medullary widening
3. Chronic hemolytic anaemia
4. Hereditary spherocytosis
5. Corrected retic count = reticulocyte % × (patient Hct/normal Hct)
RADIOLOGY71
Radiography
Radiography1

This barium enema demonstrates a tapered transition zone to a normal-


caliber colon.

Hirschsprung disease.
Radiography2
Radiography3

Barium enema - Coil (SPRING SIGN)

D x Intussusception

Rx Surgery (reduction or resection)


Radiography4
Radiography5
1) What is the radiological investigation

2) What sign is demonstrated?

3) What is the diagnosis?

4) What is the commonest age group in which the following condition occurs?

5) What are the other conditions associated with this abnormality?

Answer
1. Upper Gl barium meal study
2. Corkscrew duodenum
3. Malrotation with a midgut volvulus
4. Usually newborns and young infants
5. Duodenal atresia, duodenal stenosis, annular pancreas
Radiography6
1. Name the Investigation ?
2. What is the diagnosis ?
3. What are the further investigations ?

Answer:
1. MCU
2. Posterior urethral Valve(dilatation of prostatic urethra/distal focal stenosis)
3. DTPA (diethylene tetra amine pentoic acid) Function
4. DMSA (dimercapto succinic acid) -scarring
Radiography7
1. what is the name of this test?
2. what is your next test?

Answer
1. VCUG
2. DMSA

VCUG can Dx
 PUV in male and demonstrating VUR
 Detect urethral structure or obstruction

VUR grade
 GI ureter only
 GII pelvic and calyces but no dilatation
 GIII mild dilation, calyces blunted .
 GIV mild dilation, calyces blunted , mod. ureter dilatation
 GV dilated and tortuous ureters, along with clubbing of most calyces +IRR
The severity of reflux is classified as follows:

 Mild – Grades I and II


 Moderate – Grade III
 Severe – Grades IV and V

Management
Management based on long-term, low-dose antibiotic prophylaxis
In children who have normal bladder control and no symptoms of detrusor
dysfunction, and who have been free of infection on prophylaxis, evidence
suggests that there is little benefit from continuing prophylaxis beyond 5 years old
Screening of siblings or offspring of index children or parents should be considered

Clinical evaluation
Assessment of reflux
the only tests that routinely and reliably detect reflux are
 VCUG
 nuclear cystography.

an initial VCUG provides better anatomic details regarding reflux, including the
presence or absence of periureteral diverticuli, ureteral duplication and
abnormalities of the bladder, such as trabeculations or urethral obstruction
nuclear cystography reliably detects all grades

Assessment of renal scars


DMSA scintigraphy has been found to be a more sensitive study than intravenous
pyelography (IVP) for the detection of reflux nephropathy. in addition, the scan
can detect changes of acute pyelonephritis with greater sensitivity and specificity
than CT MRI, or ultrasonography
Mercaptoacetyltriglycine (MAG3) has also been used for renal imaging and has an
improved capability inthe detection of renal scarring compared to IVP

Controlled studies show no benefit for surgery over conservative management for G I-III
Significant spontaneous resolution rate; less likely in grades IV and V
Surgery may be indicated where prophylaxis fails to control infection and where there is
progressive reflux nephropathy;

Operative management
 Endoscopic treatment of VUR by injection of synthetic material at ureteric orifice
 Endoscopical ureteral re-implantation
Ultrasonography
Ultrasonography 1
1. What is the clinical diagnosis?
2. Mention four causes for the same

ANSWER

1. Intaventricular hemorrhage
2. Prematurity- germinal matrix bleed
Haemorrhagiic disease of the newborn
Neonatal ITP
Trauma/ Child abuse
Ultrasonography 2

G I germinal matrix hrg without ventricle dilatation


GII germinal matrix hrg extend to ventricle without dilatation
GIII germinal matrix hrg extend to ventricle with dilatation
GIV inraparenchymal hrg /PVL/porencephalic cyst
Ultrasonography 3
A premature baby was ventilated and on 2nd day had convulsions. His investigation done
which is shown here

1. Identify and describe the investigation


2. Spot the diagnosis with grade
3. What are preventive measures?
4. What is the commonest neurological sequel?
5. What is the commonest Opthalmological sequel?

Answers

1. Cranial USG showing cystic PVL


2. Grade 2 PVL
3. Early interventions , maintain normal cerebral perfusion
4. Spastic diparesis
5. Strabismus, nystagmus, ROP
Ultrasonography 4
1. What is the diagnosis?
2. What is the electrolyte imbalance seen?

ANSWER

1. Congenital hypertrophic pyloric stenosis


2. Hypochloremic metabolic alkalosis

Ultrasound examination is used to confirm the diagnosis if no mass is felt


with thickness > 4mm and length > 14mm that confirm Dx .
Ultrasonography 5
1. Describe:
2. Diagnosis:
3. Treatment :

ANSWER

1. USG abdomen
2. Intussusception
3. emergency hydrostatic reduction( if not in shock) If unsuccessful
..surgery

classic target sign of an intussusceptum inside an intussuscipiens


Longitudinal ultrasound of a patient with suspected intussusception shows the
layered bowel walls of the outer and inner loop, the intussuscipiens, and the
intussusceptum
Isotope scan 1
Isotope scan2
1. What is this modality of investigation?
2. Name three common indications

Answer::
1.Radionuclide bone scan
2.Indication
– Metastasis
– Osteomyelitis
– Stress fracture
– NAI
Isotope scan3

1. What is this modality of investigation?


2. Name one common indication .
Answer :
1• Hepatobiliary scintigraphy
2• Indication– Biliary atresia /shows good uptake by the liver,but no excretion.
Isotope scan4
Isotope scan5
1. What is the diagnosis in this DTPA scan?

2. What is the full form of DTPA?

ANSWERS

1. Absent excretory function in left kidney

2. DTPA- Diethylene triamine penta acetic acid


Isotope scan6
1. Identify

2. Diagnosis

3. Time needed between injection & imaging.

4. Four indications

Answer
1. Technetium99m DMSA (Di mercapto succinic acid) Scan
2. Diagnosis : Ectopic Rt Kidney
3. Time needed between injection & imaging: 3 hours
4. Four indications:
1. Assessment of Renal Scarring
2. Urinary Tract Infection
3. Pyelonephritis
4. Renal infarct
5. Horseshoe kidney
6. Ectopic kidney
Isotope scan7
1. Identify
2. What information do we get?
3. Time needed between injection & imaging.
4. Four indications

Answer
1. Identify: DTPA (99mTc-diethylene triamine pentaacetic acid) Scan
2. information do we get?
-Renal blood flow,
-GFR,
-Tubular function
-Urinary excretion.
3. Time needed , Immediate images of the kidneys are taken as the
injection enters the body.
4. Four indications:
-Assessment of GFR, -Differential renal function
-Acute renal failure -Chronic renal failure
-Acute & chronic rejection -Screening patients with suspected renal HTN
-Obstruction -Hydronephrosis
Isotope scan8
2 yr old boy brought with H/o intermittent painless rectal bleeding for last few months.
The stool is described as brick colour or currant jelly colour. There is anaemia. Following
study was performed.

1)What is the test performed?

2)Name the isotope used in the test.

3)Identify the dark areas on the film ,what Dx

4)How do you enhance the yield of this test?

5)What is the most appropriate next step?

5)Mention 3 less common presentations of the disease?

Answer
1)Meckels Radionuceotide scan
2)Technetium-99m(pertechnetate)
3) a)Stomach Ectopic acid secreting mucosa in meckel's diverticulum
b) Urinary bladder
c) D x - Meckel diverticulum
4)The uptake can be enhanced with various agents, including ranitidine, glucagon, gastrin.
5)Surgicalconsultation should be obtained for resection of the
6)Less common presentations include Meckel diverticulitis
Appendicitis , intestinal obstruction from intussusception or herniation, and (rarely)
perforation from an ingested foreign body trapped in the diverticulum.
CT SCAN
MRI
SCAN1
SCAN2
5-year-old boy lethargic since awakening this morning. complaining of
headaches and has been having morning emesis for several weeks. also
reports that he has become increasingly"clumsy" over the past 4 to 6 weeks.
On physical examination, the patient is difficult to arouse; has sluggish
pupillary responses; and has rapid, deep, sustained breaths at a rate of 35
breaths/min. His C/T scan is shown below

1- What is the most likely cause for this clinical presentation?


2- What is the best step in the initial management of this patient?

Answer
1- underlying brain tumor a large posterior fossa mass (medulloblastoma)
with associated hemorrhage, obstruction of the 4th ventricle, and dilation of
the lateral and 3rd ventricles.

2- Emergent intubation and control of the airway is indicated before obtaining


diagnostic imaging because the child is at significant risk for brainstem
herniation and respiratory arrest.
SCAN3
1. Describe this CT Scan.
2. What is the diagnosis?

Answer

1. The CT scan above demonstrates the mass effect on the main structures
of the brain with Midline shift and the ventricles being compressed.
2. Extradural /epidural hematoma (concave shape)
SCAN4
A 3-month-old infant with fussiness and decreased alertness ,seizure. Physical
and a bulging fontanelle.

1-What is the most likely diagnosis ,how can confirm?

2- Is angiography necessary for the diagnosis?

Answer
1-Subdural hematoma(convex) “shaken baby” or “shaken impact syndrome.

Retinal examination need to identify retinal hemorrhages to support Dx.

2- Subdural hemorrhages result from trauma to veins traversing the subdural


space, making angiography unnecessary.
SCAN5

1. What is the clinical diagnosis?


2. Mention two points seen in this CT to justify your Diagnosis ?
3. What is the immediate management?
4. Name the complication seen in the CT?

Answer
1. Cerebral abscess - Right fronto parietal region
2. Hypodense lesion measuring about 3 cm x 2cm Ring enhancement - larger area
differentiates this from granuloma and irregularity
3. Surgical drain
4. Ventriculitis right lateral ventricle

In MRI T2 weighted images indicate increased signal intensity and


demonstrates an abscess capsule.
SCAN6

1. What is the lesion?


1. 3 Differential diagnosis
2. 3 differentiating features
3. Give two management modalities.
4. What complication occur during management & how can it be
prevented?

Answer
1. Ring enhancing lesion
2. NCC, tuberculoma, toxoplasma,abscess, tumor
3. Peripheral vs central
single vs multiple
perilesional edema
smooth vs irregular
Scolex seen
Wall thickness
Intensity of cystic material

4.NCC Anticonvulsants and Albendazole


5.Increase in Cerebral Edema by Use steroid cover
SCAN7

What Diagnosis?

Right parietal lobe edema with midline shift.


SCAN8

What Diagnosis?

Intracranial hemorrhage right temporoparietal region with midline


shift. Seen in Late onset vitamin K dependant bleeding (late onset
HDON)
SCAN9
2. What is the lesion?
3. What Diagnosis?
4. 3. List 2 possible clinical findings accompany the likely diagnosis.

Answer

1. This is a CT scan of the brain.


The striking abnormalities are ventriculomegaly and periventricular
calcification
2. congenital cytomegalovirus need Urine sent for CMV detection by PCR
and blood for serology for CMV IgM. This has a sensitivity of
approximately 70%.
3. 3. Hepatosplenomegaly, sensorineural hearing loss, chorioretinitis,
thrombocytopenia, and encephalopathy.
SCAN10
2yrs old child with seizure, behavioral problems, skin abnormalities, MRI done
1. Describe findings ?
2. Identify diagnosis?
3. What are criteria?
4. What is inheritance?
5. Associated Kidney problem?
6. What are chart eristic CNS presentation
7. How u follow up these cases?

Answer
1. Axial CT shows multiple foci of periventricular hyperattenuation consistent
with subependymal calcifications of tuberous sclerosis
2. Tuberous sclerosis
3. Major / minor criteria
4. AD
5. Angiomyolipoma
6. Epilepsy/ cognitive impairment
7. F/u examination
Cranial MRI 1-3 yrs
Abdomen USG/CT/MRI renal 1-3 yrs
Neurodevelopment assessment
SCAN11

4 year old boy with the following photo and CT.


1. Identify syndrome
2. Describe the CT
3. Mode of inheritance

Answer

1. Sturge-Weber Syndrome
2. Axial non enhanced CT scan shows left hemiatrophy of the cerebral
cortex and typical gyral calcification
3. Sporadic
SCAN12

1. Identify and describe the findings.


2.What is probable diagnosis ?
3. What is the treatment?

Answer
1. CT scan showing Ventricular dilation
2. Hydrocephalus
3. Medical Acteazolamide, furosemide and surgical VP shunt
SCAN13
1.Identify ?

2.Describe findings ?

3.Associated syndrome?

Answer

1.Lissencephaly

2.Findings

› Absence of cerebral convolutions

› Maldeveloped sylvian fissures

› Enlarged ventricles.

3. Miller-Dieker syndrome.
SCAN14
1.Identify ?

2.Describe findings ?

3.whats Dx syndrome?

Answer

1. Transillumination demonstrates a posterior fossa cyst, bulging occiput,


prominent scalp veins, and enlargement of the head.

CT scan shows a posterior fossa cyst, a small cerebellar remnant, and


associated hydrocephalus

2. Dandy-Walker malformation.
SCAN15

T2 MRI scan

The scan shows severe hydrocephalus with very marked reduction of the
cortical mantle.
The inter-ventricular septum is absent.
The diagnosis in this case is Dandy–Walker malformation.
This is associated with severe hydrocephalus, as seen in this scan.

Rx Medical Acteazolamide, furosemide and surgical VP shunt


SCAN16
1. What are the MRI findings ?
2. What is the DD
3. What further inv will you do on the CSF to confirm the diagnosis ?

MRI BRAIN Protocol Sequences: Axial Tl, T2W& DWI; Coronal FLAIR; Post- contrast TIW
axial, sagittal and coronal planes imaging.

Finding Multiple large foci of hyperintensity are seen in the supra tentorial white matter,
primarily involving both temporoparietal lobes (L>R), the occipital lobes(R>L) and the
high frontal lobes on T2W and FLAIR images, appearing hypointense on TlWI. The corpus
cauosum, basal ganglia, the thalami, the internal capsules, the infra tentorial
compartment and the visualised cord are spared. The gray matter is involved only in the
temporal lobes. There is no mass effect. On DWI there is sub acute restricted diffusion in
the affected areas.

IMPRESSION:Multifocal supratentorial demyelination suggest the possibilities of-

(a) Acute disseminated encephalomyelitis(ADEM)

(b) Acute MS
SCAN17
1. Diagnosis

2. By what gestational age would this defect occur?

3. This can be prevented in subsequent pregnancies by intake of Folic acid. Folic acid
should be taken in what dose and started when?

Answers

1. Occipital Encephalocele

2. 26 days post conception

3. Folic acid /Dose: 0.4mg/day Periconceptional period


SCAN18

1. What is the finding?


2. What is the diagnosis?
3. What causes this finding on the MRI?
4. What is the mode of inheritance?
5. Give two clinical features of this condition.

Answers

1. Molar tooth sign


2. Joubert’s syndrome
3. Absence of decussation of superior cerebellar peduncles
4. Autosomal recessive
5. Ataxia, irregular respirations, hypotonia, abnormal eye movements
,retinitis pigmentosa, polydactyly, nephronopthisis.
CLASSIFIED UNDER CILIOPATHIES
SCAN19
Our patient is a 17 yr old female with history of recurrent seizures. Admitted in PICU with
stroke. She was evaluated for same.

1. Identify and describe the investigation?


2. What is the diagnosis?
3. What other disorder are associated with this disease?
4. What is the treatment of choice?
5. What are syndromes associated with contralateral hemiplgia in children?

Answer

1. a)MRI T1 weighted show extensive collateralization of vessels in region of circle of


willis.

b) MRA reveals reduced luminal calibre of left internal carotid artery in petrous,
cavernous and supraclinoid segments.

2.Moya moya disease.

3.Down syndrome, neurofibromatosis, or sickle cell disease

4.Direct superficial temporal artery (STA) to middle cerebral artery (MCA) bypass is
considered the treatment of choice.

5.Weber syndrome. Benedict syndrome .millard gublar syndrome.


SCAN20

6 months old female child, FTND,has asymmetric infantile spasms, developmental


retardation and choroidal lacunae.

MRI clinches the diagnosis.

1. What is the diagnosis?


2. What is the MRI picture?
3. What is the inheritance pattern?

Answer
1. Aicardi syndrome
2. Corpus callosum aPosterior choroidal cysts,choroidal lacunae
3. X linked Dominant
SCAN21
2 yrs old child born prematurely [28wks] comes with complaint of delayed achievement
of motor milestones.

Examination shows hypertonia with brisk reflexes in all 4 limbs more marked in lower
limbs.

1. What is the diagnosis? Define.


2. What is the MRI picture?
3. What is the treatment and when do you start it?
4. Name 2 conditions this child will be at risk for?

Answer
1. Spastic Diplegia. Increased tone in all 4 extremities and face. LL>UL> fac
2. Periventricular leukomalacia
3. Early intervention
Learning disabilities,
Visual concerns
Epilepsy
4. Delayed mental development
Behavior concerns
SCAN22
A full term ,male child develops jaundice on day 3 of life, (S. bilirubin -34 mg%)and
undergoes an exchange transfusion .

1. What is the immediate complication likely to occur?


2. Where is the anatomical abnormality?
3. What are the long term complications?
4. Name 1 investigations you would insist on after discharge?

Answer
1. Kernicterus
2. Globus pallidus, dentate nucleus, cerebellar vermis, cochlear nuclei
3. Choreoathetoid cerebral palsy, dystonic / dyskinetic CP, Sensorineural deafness,
4. Audiometry testing
SCAN23
A child with apparently normal birth history :MS- social smile at 3 mths, Sitting at 8
months, rt hand preference at 9 mths, walking independently at 1.6 yrs.

1. What is your diagnosis?


2. What investigation will you do?
3. What relevant maternal history will you ask for?

Answer
1. Congenital hemiplegia
2. MRI brain
3. Maternal injury, threatened abortion,APC resistance, coagulation problem
.
SCAN24

1. What is the finding?


2. What is the diagnosis?

1-This MRI scan shows agenesis of the corpus callosum. The lateral ventricles
are abnormal and crescentic in shape. They are shifted laterally resulting in
the formation of a large midline interhemispheric subarachnoid space.
The abnormal shape is due to deformation by fibres of the cerebral
hemisphere that was meant to cross in the corpus callosum, but due to the
agenesis, run longitudinally as the bundles of Probst.

2. Agenesis of the corpus callosum can be an isolated finding but can be


associated with other structural brain anomalies. It is seen in Aicardi, Apert’s,
Smith–Lemli–Opitz, Goldenhaar, Fryns ’,Meckel–Gruber, Zellweger’s and
Walker–Warburg syndromes.
It is also seen with inborn errors of metabolism and in fetal alcohol syndrome.
It is therefore important not only to look for other anomalies on the MRI scan,
but to look at the infant for anomalies in other systems.
SCAN25

1. Identify the clinical condition

2. What is the sign seen in the imaging study?

3. How will you treat the condition?

Answer

1. Pyothorax
2. Split pleura sign
3. IV antibiotics
Intercostal drainage
VATS
SCAN26

1. Describe what you see


2. What is the diagnosis?

Answer
1. large, homogenous mass is seen on the right side of the abdomen, arise
from the right kidney
2. Wilms tumor
SCAN27

1. Describe the CT findings.


2. Give DD

Answer
1. large, inhomogenous mass is seen on the right side of the abdomen,
compressing the right kidney posteriorly. The mass did not appear to arise
from the right adrenal gland or right kidney
2. DD
 Neuroblastoma
 Wilms tumor
 Lymphoma
 Soft tissue sarcoma
 Pheochromocytoma
SCAN28
1. Describe:
2. Diagnosis:
3. Diagnostic Criteria:

Answer
1. MRI axial image of thigh
2. Dermatomyositis
3. Classic Rash + 3 of the following
 Weakness
 Muscle enzyme elevation
 EMG changes
 Muscle Biopsy

You might also like